CASOS-EGEL

August 7, 2018 | Author: Karla S. Reséndiz | Category: Asthma, Childbirth, Pneumonia, Pregnancy, Pelvis
Share Embed Donate


Short Description

Download CASOS-EGEL...

Description

Caso 1.1.- la conjuntivitis hemorrágica aguda es ocasionada por 1 virus de la familia de: a) picornavirus b) adenovirus c) rotavirus d) flavivirus e) retrovirus CC. inicio súbito, bilateral, caracterizado principalmente por dolor ocular intenso, fotofobia, visión borrosa, epífora, hiperemia conjuntival con hemorragias subconjuntivales de tamaño variable, y edema y quemosis  palpebrales. El cuadro se presenta también con adenomegalias preauriculares y secreción serosa ocular  que pos poster terior iormen mente te se pue puede de vol volver ver muc mucopu opurul rulent enta a en cas caso o de inf infecc ección ión bac bacter terian iana a sec secund undari aria. a. Generalmente aparece precedida por un cuadro infeccioso de vías aéreas superiores Etio.con mayor frecuencia son el virus Coxackie A 24 y el Enterovirus 70. Con respecto a los adenovirus, estos son una de las principales causas de conjuntivitis folicular y de queratoconjuntivitis. Dx clínico Tx medidas grales, compresas frias, lagrimas artificiales, vasoconstrictores.

Caso 2.Paciente masculino de 30 años de edad. Obrero, que después de esmentar una puerta refiere sensación de cuerpo extraño, lagrimeo y dolor ocular derecho. 2.- el sitio mas frecuente donde se localiza el cuerpo extraño es: a) conjuntiva b) cornea c) parpados d) cristalino e) retina 3.- el tratamiento indicado será: a) lavado ocular b) antibióticos tópicos c) corticoides tópicos d) vasoconstrictores tópicos e) extracción de cuerpo extraño 4.- además del tratamiento anterior ¿Qué mas indicaría? a) corticoides locales b) vasoconstrictores tópicos y sistémicos c) antibióticos tópicos mas aposito d) corticoides locales mas aposito e) analgésicos Caso 3.Una mujer de 68 años de edad es hospitalizada con historia de nauseas y vomito en las ultimas 48 horas asociados con cefalea del lado izquierdo predominantemente en el ojo. Ella no recuerda otros episodios similares. La tensión arterial es de 115/78 mmHg y el pulso de 96X’. A la exploración física el ojo izquierdo se aprecia mas pequeño, la pupila con dilatación e hiperemia hiperemia conjuntival. conjuntival. La visión visión es 20/20 en el ojo derecho y 20/400 en ojo izquierdo. izquierdo. La pupila derecha reacciona normalmente a la luz, pero la izquierda no es reactiva. 5.- el diagnostico más probable en esta paciente es: a) iritis aguda b) uveítis aguda c) arteritis temporal d) glaucoma agudo e) migraña común 6.- es la prueba diagnostica mas útil en esta paciente a) radiografía de cráneo b) tonometria c) angiografía carotidea d) angiofluorografia e) oftalmoscopia Caso 4.-

1

Un niño de 6 años de edad es atendido por un medico, la madre refiere fiebre no cuantificada acompañada de escalofríos, ataque al estado general, mialgias y artralgias, dolor faringeo, odinofagia, cefalea y vomito en una o casión A la exploración física se observa hipertrofia amigdalina grado III, amígdalas hiperemicas y con caseum, linfadenopatia cervical, temperatura de 38oC 7.- el agente etiológico más común en este padecimiento es: a) estafilococos auerus b) estreptococo beta hemolítico del grupo A c) rinovirus d) diplococo e) estreptococo beta hemolítico del grupo B 8.- epidemiologicamente que etapa de la vida afecta más a) preescolar b) escolar c) adolescente d) neonato e) lactante 9.- el fármaco de primera elección o erradicación en este padecimiento es: a) trimetropim con sulfametoxasol b) penicilina c) ampicilina d) eritromicina e) amikacina 10.- estos pacientes tienen como complicación frecuente: a) glomerulonefritis b) neumonía c) endocarditis d) meningitis e) fiebre reumática Caso 5.paciente femenina de 20 años de edad, refiere desde hace 2 días con odinofagia, cefalea, rino rinorr rrea ea hial hialin ina, a, tos tos prod produc ucti tiva va en acce acceso sos, s, hiper hiperte termi rmia a de hast hasta a 38º 38º C, difi dificu cult ltad ad respiratoria. A la exploración física con tensión arterial de 110/ 80 mmHg, frecuencia respiratoria 32 por minuto, frecuencia cardiaca 100X’, temperatura de 38º C se observa tiraje intercostal y a la auscultaci auscultación ón con campos pulmonares pulmonares se escuchan escuchan sibilanci sibilancias as diseminada diseminadas s y espiración espiración prolongada no hay cianosis. 11.- el cuadro clínico es sugestivo de: a) neumonía lobar b) neumonía atípica c) laringotraqueitis d) crisis asmáticas e) bronquitis aguda 12.- el manejo seria con: a) agonista beta b) agonistas alfa c) antagonistas beta d) antagonistas gamma e) antibióticos 13.- la dosis de impregnación de aminofilina es: a) 0.6mg/Kg en 20-30 minutos b) 0.6 mg/Kg/hora c) 6 mg/Kg en bolo directo d) 6 mg/Kg en 20-30 minutos e) 6 mg/-Kg en 1 hora

2

Un niño de 6 años de edad es atendido por un medico, la madre refiere fiebre no cuantificada acompañada de escalofríos, ataque al estado general, mialgias y artralgias, dolor faringeo, odinofagia, cefalea y vomito en una o casión A la exploración física se observa hipertrofia amigdalina grado III, amígdalas hiperemicas y con caseum, linfadenopatia cervical, temperatura de 38oC 7.- el agente etiológico más común en este padecimiento es: a) estafilococos auerus b) estreptococo beta hemolítico del grupo A c) rinovirus d) diplococo e) estreptococo beta hemolítico del grupo B 8.- epidemiologicamente que etapa de la vida afecta más a) preescolar b) escolar c) adolescente d) neonato e) lactante 9.- el fármaco de primera elección o erradicación en este padecimiento es: a) trimetropim con sulfametoxasol b) penicilina c) ampicilina d) eritromicina e) amikacina 10.- estos pacientes tienen como complicación frecuente: a) glomerulonefritis b) neumonía c) endocarditis d) meningitis e) fiebre reumática Caso 5.paciente femenina de 20 años de edad, refiere desde hace 2 días con odinofagia, cefalea, rino rinorr rrea ea hial hialin ina, a, tos tos prod produc ucti tiva va en acce acceso sos, s, hiper hiperte termi rmia a de hast hasta a 38º 38º C, difi dificu cult ltad ad respiratoria. A la exploración física con tensión arterial de 110/ 80 mmHg, frecuencia respiratoria 32 por minuto, frecuencia cardiaca 100X’, temperatura de 38º C se observa tiraje intercostal y a la auscultaci auscultación ón con campos pulmonares pulmonares se escuchan escuchan sibilanci sibilancias as diseminada diseminadas s y espiración espiración prolongada no hay cianosis. 11.- el cuadro clínico es sugestivo de: a) neumonía lobar b) neumonía atípica c) laringotraqueitis d) crisis asmáticas e) bronquitis aguda 12.- el manejo seria con: a) agonista beta b) agonistas alfa c) antagonistas beta d) antagonistas gamma e) antibióticos 13.- la dosis de impregnación de aminofilina es: a) 0.6mg/Kg en 20-30 minutos b) 0.6 mg/Kg/hora c) 6 mg/Kg en bolo directo d) 6 mg/Kg en 20-30 minutos e) 6 mg/-Kg en 1 hora

2

14.- los efectos tóxicos mas frecuentes de la aminofilina son: a) cefalalgia y alucinaciones visuales b) nauseas y vomito c) convulsiones y arritmias ventriculares d) vomito y deshidratación e) taquipnea, disnea 15.- usted indica intubación endotraqueal y apoyo mecánico ventilatorio con la presencia de: a) patrón gasometrico, cianosis, taquipnea y alcalosis respiratoria b) patrón gasometrico, alcalosis respiratoria, taquicardia y taquipnea c) patrón gasometrico, acidosis metabólica, taquicardia y cianosis d) patrón gasometrico, acidosis e insuficiencia respiratoria y cansancio físico e) patrón gasometrico, taquicardia, vomito y acidosis mixta. Caso 6.Paciente femenina de 41 años de edad que ingresa a urgencias con padecimiento de 24 horas de evolución caracterizado caracterizado por insuficiencia respiratoria progresiva y persistente hace 2 meses fue tratada del mismo problema respondiendo favorablemente con la administración de esteroides por vía oral. A la exploración física con tensión arterial de 90/59 mmHg, frecuencia cardiaca de 150 por minuto, temperatura de 38º C y 14 respiraciones por minuto. Se encuentran periodos de broncospasmo y aumento del trabajo respiratorio. Se administra hidrocortisona, salbutamol y dopamina, sin obtener respuesta favorable por lo que se decide a intubación endotraqueal para ventilación asistida. 16.- diagnostico de este paciente: a) asma bronquial b) bronquitis crónica c) neumonía de focos múltiples d) estado asmático e) enfisema pulmonar La exacerbación asmática, en un tiempo llamado status asmático, hace que el paciente acuda a solicitar atención medica. Se trata de episodios agudos o subagudos de un deterioro rápidamente progresivo de la respiración, tos, sibilancias, y retracciones y tiros torácicos, o combinación de estos síntomas. La exacerbación está caracterizada por un disminución del flujo respiratorio que puede ser documentado rápida y prácticamente, por volumen expiratorio forzado en un segundo (FEV1) o pico máximo de flujo expiratorio (PEF). El objetivo de realizar estas mediciones es indicar la severidad de la exacerbación que da la severidad de los síntomas. Criterios dx de asma grave Duración prolongada de los síntomas. Progresión a pesar de tratamiento correcto. Disnea que impide dormir. Disnea que impide hablar. Empleo de musculatura respiratoria accesoria. Taquicardia >120 lpm. Taquipnea >35 rpm. Pulso paradójico >15 mm Hg. FEV1 Pico de flujo espiratorio forzado Aumento o descenso de paCO2.

Criterios de intubación en asma grave Taquipnea >40 rpm. · Aumento del pulso paradójico. · Caída del pulso paradójico en paciente exhausto. · Alteración del nivel de consciencia. · Imposibilidad de hablar. · Barotrauma. · Acidosis láctica persistente. · Diaforesis profusa. · Silencio pulmonar a pesar de esfuerzos respiratorios. · Elevación de paCO2 con progresión de síntomas.

17.- además de la posibilidad diagnostica de ingreso, ¿en que otro padecimiento se debe pensar? a) infección bronquial agregada b) neumonía lobar bilateral c) absceso pulmonar lobar d) ruptura de bula subpleural e) insuficiencia cardiaca congestiva

3

Después de la medicación especifica, disminuye la fiebre pero inmediatamente después se regist reg istra ra diures diuresis is de 2.500ml 2.500ml en 24 horas, horas, deshid deshidrat rataci ación, ón, cuerpo cuerpos s cetonic cetonicos os en orina orina y glucemia de 320. 18.- debemos pensar en: a) diabetes mellitus b) diabetes secundaria c) estado hiperosmolar d) reflejo de cushing e) diabetes insípida 19.- los factores que contribuyen a la poliuria, deshidratación y cetonuria son: a) salbutamol y fiebre b) acidosis metabólica e hipernatremia c) sudoración e hipertermia d) esteroides e infección e) fiebre e hipercloremia 20.- en el tratamiento de urgencias las indicaciones indispensables son: a) restricción de líquidos e hipoglucemiantes o ral b) dieta baja en proteínas y baja de lípidos c) digitalico y oxigeno por mascara d) insulina rápida y antibiótico e) antibiótico e insulina intermedia Caso 7.Niño de 12 años de edad que inicia hace tres días con fiebre, cefalea, malestar general. Disfag Disfagia, ia, dolor faring faringeo, eo, anorex anorexia ia y vomito vomito en las ultimas ultimas 12 horas horas reapar reaparece ece un rash rash cutáneo cutáneo consistent consistente e en papulas papulas eritematosas eritematosas,, difusas, difusas, puntiformes puntiformes localizadas localizadas en cuello, cuello, pliegu pliegues es axilar axilares, es, fosas fosas antero anterocub cubita itales les e ingle, ingle, la faring faringe e y amígda amígdalas las están están rojas rojas y edematosas. 21.- este cuadro clínico indica la presencia de: a) dermatitis atopica b) sarampión c) rubéola d) fiebre escarlatina e) varicela 22.- el manejo adecuado de esta condición será: a) ribavirina b) dar tratamiento sintomático solamente c) hospitalizarlo inmediatamente d) antibióticos antiestreptococo e) dar un antihistamínico y tratar la fiebre Caso 8.Un niño de un año y medio de edad, previamente asintomático despierta en la madrugada con falta de aire, discreta cianosis, respiración estertorosa y tos seca. Es llevado a urgencias encontrándos encontrándose e febril, febril, con taquipnea taquipnea discreta, discreta, estridor inspirator inspiratorio, io, llanto disfonico, tos perr perruna una y pulm pulmon ones es con con este estert rtore ores s grues gruesos. os. La biome biometr tría ía hemat hematic ica a reve revela la disc discre reta ta leucocitosis. 23.- el diagnostico que usted haría en este caso es: a) Bronquitis b) Bronqiolitis c) Rinofaringitis d) Laringotraqueitis e) Neumonia 24.- ¿cual de las medidas siguientes es más útil en el tratamiento inicial? a) dexametasona IM b) eritromicina c) acetominofem y baño

4

d) oxigeno e) ambiente húmedo Caso 9.En una locali localidad dad de campec campeche he se realiz realizo o un estudi estudio o sobre sobre la incide incidenci ncia a de tuberc tuberculos ulosis is pulmonar. Todos los individuos de la localidad fueron examinados en 2 encuestas (1990 y 1991) y el número de casos nuevos encontrados, al aplicar la segunda encuesta fue utilizado para determinar la incidencia. Los resultados del estudio están descritos en la tabla siguiente.

Categoría de la vivienda en la 1ª Encuesta (1990) Con casos positivos Sin casos positivo

numero de personas

500 10,000

# de casos nuevos 1990-1991 10 10

INCIDENCIA ACUMULADA: (#CASOs nuevos de enfermos en el e l periodo/#personas libres de la enfermedad en la población 25.- ¿Cuál es la incidencia de tuberculosis por 1000 habitantes en las viviendas que tuvieron un caso positivo, durante la 1ª encuesta a) 001 b) 0.02 se divide 10/500. c) 200 d) 1000 e) 20.00 RIESGO RELATIVO :

es la proporción entre del riesgo de la población expuesta con respecto a la población no expuesta. RR:riesgo en pob expuesta/riesgo pob no expuesta x100  26.- ¿Cuál es el riesgo relativo de adquirir tuberculosis en las viviendas con casos positivos en comparación con las viviendas donde no hay casos? a) 0.05 b) 0.50 c) 2.00 d) 10.00 e) 20.00 Caso 10.Un abogado de 57 años de edad tiene antecedentes de tos de 10 meses de evolución así  como disnea progresiva con el ejercicio. Desde hace 20 años dejo de fumar. A la exploración física se observa aparentemente saludable con un pulso de 84 por minuto la tensión arterial de 142/90 mmHg, frecuencia respiratoria de 18 por minuto y temperatura de 37º C había hipocratismo digital 27.- el diagnostico más probable será: a) carcinoma broncogenico b) enfisema pulmonar c) tuberculosis pulmonar d) asma bronquial d) silicosis Caso 11.Un niño de 3 años ha estado padeciendo de tos severa desde el nacimiento expulsando esputo esputo adherente adherente y espeso espeso con gran dificulta dificultad. d. Presenta Presenta también episodios episodios febriles con dificultad respiratoria y cianosis discreta. 28.- el diagnostico presupsial en este paciente será. a) secuestro pulmonar b) tuberculosis c) fibrosis quistica d) fístula traqueoesofagica e) bronquiectasias 29.- el diagnostico se confirma mediante:

5

a) radiografía de tórax b) cloruros de sudor c) prueba para secretina de la función pancreática d) broncografia e) espirometría diferencial 30.- el tratamiento es a base de: Caso 13.Paciente femenino de 32 años de edad en puerperio fisiológico quien refiere iniciar padecimiento hace 3 días, caracterizado por presentar en miembros pélvicos derecho dolor, edema dorsiflexion dolorosa de la piel, eritema y dolor a la presión manual de los músculos gemelos, axial como signo de la cuerda positivo. 31.- con los datos anteriores la sospecha clínica corresponde a : a) tromboflebitis superficial b) tromboflebitis media c) tromboflebitis profunda d) flebitis inespecífica e) linfangitis 32.- el apoyo diagnostico adecuado es: a) venografia con material de contraste b) ultrasonido con doppler c) arteriografía d) plestimografia e) angioflouroangiografia 33.- los exámenes básicos que usted solicitaría previos al manejo medico además de la biometría hematica son: a) solamente recuento plaquetario b) electrocardiograma, radiografía de miembros pélvico derecho, colesterol serico c) fibrinogeno, recuento plaquetario. Electrocardiograma d) cuenta plaquetaria, tiempo de protrombina, tiempo parcial de protombina y radiografía de tórax e) tele de tórax, gammagrafia pulmonar 34.- el tratamiento de elección inicial considerando la lactancia es: a) acido salicílico 1g al día b) acenocomarina 8 mg vía oral c) dipindamel 75 mg vía oral d) heparina subcutánea 5000 U dosis inicial e) heparina IV 125- 150 U/Kg. 35.- la principal complicación del tratamiento anterior es: a) bacteriemia b) alargamiento del tiempo de protrombina c) hemorragia d) glomerulonefritis e) tromboembolia pulmonar Caso 14.De la revisión de las investigaciones llevadas acabo en el estudio de la hipertensión arterial sistólica gran parte corresponden a estudios que consideran variables de interés que existen en una determinada población en un momento co ncreto. 36.- estos estudios se denominan especialmente: a) transversales b) longitudinales c) epidemiológicos d) de cohortes e) prospectivos

6

37.- los estudios estadísticos en hipertensión arterial que permiten conocer satisfactoriamente el efecto de múltiples variables sobre una o más variables dependientes se denominan. a) análisis de regresión de multivariables b) análisis bivariantes c) análisis multivariantes d) transversales e) longitudinales 38.- considerando el porcentaje de incidencia de la HAS secundaria es valido: a) suponer que solo se presenta en grandes hospitales b) suponer que solo importa el tratamiento a establecer c) realizar una historia detallada a fin de efectuar el diagnostico etiológico d) enviar al paciente a hospitales de referencia e) solicitar una bacteria de estudios 39.- la principal causa de hipertensión arterial secundaria es: a) la enfermedad renal parenquimatosa b) el aldosteronismo primario c) feocromocitoma d) estenosis de arterial renal e) coartación aortica 40.- la incidencia de la hipertensión arterial esencial en México es de: a) más del 9% b) 5% c) 0.3% d) 0.1% e) menos del 0.1% 41.- el factor más significativo para el abandono del tratamiento en la HAS es: a) el nivel educativo b) el costo del tratamiento c) ser asintomático d) ser de edad joven e) negación a la resistencia de la enfermedad Caso 15.Un hombre de 48 años de edad que presento dolor en área precordial mientras leía el periódico. El dolor se acompaño de sensación de adormecimiento del brazo izquierdo y cedió en forma espontánea. A la exploración física se percibe un soplo diastólico en foco aortico. 42.- el padecimiento actual corresponde a: a) angina de pecho b) esofagitis por reflujo c) miocarditis d) pancreatitis e) osteocondritis 43.- el ECG muestra ondas Q profundas, el segmento ST con depresión y las ondas T son normales. CPK, CPK-MB y DHL son normales. Con estos datos se confirma diagnostico de: a) infarto agudo de miocardio b) tromboembolia pulmonar c) pericarditis d) angina de pecho e) miocarditis Se solicita telerradiografía de tórax encontrándose ensanchamiento de la aorta torácica y depósitos de calcio con disposición linera en la porción descendente.

7

44.- con base en los datos con que se cuenta hasta este momento, ¿Qué otro diagnostico puede establecerse? a) valvulopatia reumática b) aortitis sifilítica c) infarto de gran extensión d) artritis reumatoide e) fiebre reumática 45.- la prueba que se encuentra a favor de su diagnostico es: a) reacciones febriles positivas b) reacción lepromatosa II c) proteína C reactiva negativa d) disminución progresiva de Ig G e) Serología VDRL positiva 46.- ¿a que fase de evolucion corresponde el diagnostico que estableció de la enfermedad de base? a) precoz b) primaria c) secundaria d) terciaria e) cuaternaria Caso 16.Paciente femenino de 32 años de edad con antecedentes de fiebre reumática en la infancia que le dejo como secuela estenosis mitral. Se presenta a la consulta general con disnea de moderados esfuerzos y cansancio al realizar labores propias del hogar. Los signos vitales se encuentran dentro de los limites normales, al examen general se aprecio intolerancia para permanecer acostada y al examen segmentario se observa ingurgitación yugular grado I, soplo sistólico grado I en foco mitral, escasos estertores crepitantes en ambas bases pulmonares y abdomen distendido por crecimiento uterino secundario a 5 meses de gestación. 47.- con los datos mencionados se puede proponer el diagnostico de: a) endocarditis subaguda b) insuficiencia cardiaca izquierda c) miocardiopatia dilatada d) edema agudo pulmonar e) insuficiencia cardiaca derecha 48.- el factor etiopatologico corresponde a: a) reactivación de la fiebre reumática b) aumento de la resistencia periférica c) proceso de gestación d) insuficiencia mitral agregada e) aumento de la poscarga 49.- la presencia de estertores crepitantes se explica por: a) aumento de poscarga del ventrículo izquierdo b) hipertensión arterial pulmonar c) dilatación de la aurícula derecha d) disminución de la presión en cuña de la arteria pulmonar e) aumento de la presión hidrostática 50.- entre los datos radiológicos el que orienta más el diagnostico de: a) distensión de venas pulmonares b) crecimiento de aurícula derecha c) ensanchamiento de la aorta torácica d) agrandamiento de cavidades cardiacas e) signo del 3 invertido 51.- la indicación general en este caso será: a) uso de beta bloqueadores

8

b) reposos absoluto y dieta hipocalórica c) administración de diurético osmotico d) restricción moderada de la actividad física y los líquidos e) administración de aminas simpaticomimeticas Caso 17.A un niño de 12 años de edad en un examen de rutina se encuentra soplo sistólico en el segundo espacio intercostal izquierdo. Había un desdoblamiento amplio del segundo ruido cardiaco que no se modifica durante la respiración. El componente pulmonar era mas intenso que el comportamiento aortico. El ECG revelo hipertrofia ventricular derecha y bloqueo de rama derecha. 52.- el diagnostico más probable será: a) estenosis pulmonar b) comunicación ínter auricular c) comunicación interventricular d) estenosis aortica e) coartación de aorta 53.- la placa de radiografía de tórax probablemente mostrara: a) arterias pulmonares prominentes b) campos pulmonares oligohemicos c) arcos costales carcomidos (S de Rvester) d) cardiomegalia significativa e) disminución de la vascularizacion pulmonar 54.- es un diagnostico compactible en este caso de: a) plétora pulmonar b) dilatación de la arteria pulmonar c) aumento de tamaño de la aurícula derecha d) aumento de tamaño del ventrículo derecho e) aumento de tamaño del ventrículo izquierdo. Caso 18.Paciente masculino de 40 años de edad que acude al servicio de urgencias por cefalea universal parestesias en hemicuerpo derecho, detectándole altas cifras tensiónales de 190/120 mmHg frecuencia cardiaca de 100 por minuto y en el ECG muestra ondas sugestivas de isquemia moderada en derivaciones precordiales, por lo que inicia tratamiento con nifedipino sublingual. 55.- según el grado de hipertensión arterial a que grado corresponde: a) moderado b) grave c) II d) III e) maligna 56.- el mecanismo de acción del medicamento empleado es: a) bloqueador de receptores beta b) bloqueador de canales de calcio c) inhibidor de la ECA d) vasodilatador venoso e) agonista beta 57.- a que otro medicamento pertenece a esta familia: a) captopril b) metoprolol c) furosemida d) verapamil e) minoxidil

9

58.- dado que para el control adecuado de la tensión arterial en este paciente fue necesario utilizar 40 mg de nifedipino sublingual en un lapso d 30 minutos es posible esperar uno de los siguientes efectos adversos: a) inotropismo negativo b) asistosolia transitoria c) bradicardia d) bloqueo auriculoventricular e) cefalalgia Caso 19.Un lactante de 7 meses de edad, se esta volviendo cianótico paulatinamente prominentemente en mucosa de los labios y uñas, las escleróticas son grisáceas presenta una crisis paroxística de disnea. A la exploración física se palpa un trill sistólico y se escucha un soplo holosistolico intenso áspero y mas intenso en el borde esternal izquierdo. Presenta policetemia. La radiografía revela una silueta cardiaca semejante a un sueco las áreas hiliares están relativamente claras. 59.- el diagnostico más probable es: a) persistencia del conducto arterioso b) tetralogía de fallot c) Síndrome de woll Parkinson_ White d) transposición de grandes vasos e) comunicación interventricular amplia 60.- la cianosis se debe a: a) aorta cabalgante b) estenosis aortica c) persistencia del conducto arterioso d) atresia de la arteria pulmonar e) comunicación interventricular 61.- en el ECG se espera encontrar: a) desviación del eje a la izquierda b) desviación del eje a la izquierda e hipertrofia ventricular izquierda c) desviación del eje a la derecha e hipertrofia ventricular derecha d) desviación del eje a la izquierda e hipertrofia ventricular derecha e) desviación del eje a la derecha 62.- en este caso la complicación más probable será: a) hipoxia cerebral b) insuficiencia cardiaca congestiva c) hipertensión pulmonar d) trombosis cerebral e) congestión hepática parestesia venosa 63.- procedimiento de utilidad en el presente caso: a) operación de black taussug b) reparación quirúrgica c) invertir los grandes vasos d) cierre interventricular e) dilatación tricúspidea Caso 20.Paciente femenino de 20 años de edad que inicio su padecimiento el día de ayer con dolor abdominal tipo cólico en hipogastrio y fosa iliaca derecha acompañada de nauseas y vo mito. A la exploración física se observa intranquila con fascies de dolor actitud forzada posición antialgica, pálida bien hidratada, abdomen blando depresible, hiperestesia, hiperbaralgesia en fosa iliaca derecha, tacto rectal con ampula vacía dolor en cara lateral derecha, tension arterial de 100/60 mmHg , frecuencia cardíaca de 80 por minuto, frecuencia respiratoria de 20 por minuto, temperatura de 37.6º C. 64.- con estos datos usted puede hacer el diagnostico de:

10

a) quiste torcido de ovario b) embarazo intrauterino c) apendicitis aguda d) colon irritable e) diverticulitis aguda 65.- el datos de laboratorio que corresponde a este diagnostico es: a) monocitosis b) leucocitosis c) eosinofilia d) linfocitosis e) neutropenia 66.- se debe hacer diagnostico diferencial con: a) embarazo tubario b) colon irritable c) diverticulitis aguda d) litiasis ureteral e) torsión de quiste de ovario 67.- a la exploración física usted espera encontrar los signos de: a) romemberg b) psoas obturador c) psoas d) obturador e) giordano, mac burney 68.- en una placa de abdomen esperaria encontrar a) imagen de vidrio despulido b) edema interesa c) psoas visible d) niveles hidroareos e) dilatación de asas 69.- la complicación que se presenta con mayor frecuencia es: a) choque hipovolemico b) dolor de mayor intensidad c) peritonitis d) adherencias por inflamación e) oclusión intestinal 70.- la etiología de esta patología es: a) quiste con pediculo largo b) implantación de huevo en trompa de Falopio c) origen bacteriano d) obstrucción del apéndice e) obstrucción de un divertículo Caso 21.Paciente femenino de 34 años de edad que acude al servicio de urgencias por padecer desde hace 4 semanas ataque al estado general, hiporexia, nauseas y vomito, hace una semana se agrego tinte amarillento de la piel y escleróticas, orina oscuro y dolor a nivel del hipogastrio derecho hace 3 meses fue sometida a un procedimiento quirúrgico (histerectomía) en esa ocasión la paciente fue anestesiada con halotano y transfundida con 2 paquetes globulares: 71.- el cuadro clínico corresponde a: a) hepatitis viral tipo A b) hepatitis viral tipo B c) hepatitis química d) hepatitis C e) hepatitis E

11

72.- de acuerdo a su diagnostico ¿Cuál es el periodo promedio habitual de incubación y presentación clínica de la enfermedad? a) 7 a 14 días b) 15 a 21 días c) 22 a 45 días d) 60 a 90 días e) se ignora 73.- el dato de laboratorio más útil para la corrobacion de su diagnostico es: a) Ig M antivirus de la hepatitis A b) Ig M antivirus de la hepatitis B c) bilirrubinemia d) Ig G antivirus de la hepatitis B e) transaminasemia Caso 22.Paciente femenino de 9 años de edad que inicio su padecimiento hace una semana caracterizado por adinamia, anorexia, nauseas y vomito, calosfríos, cefalea, dolor torácico y abdominal a la exploración física se le encuentra decaída con actitud libremente escogida, mucosas discretamente secas con dolor retroesternal y basal derecha, a la percusión de caja torácica, así como dolor de epigastrio a la palpación. Temperatura de 38º C, frecuencia respiratoria de 20 por minuto y frecuencia cardiaca de 100 por minuto. 74.- con los datos proporcionados, usted puede realizar el diagnostico siguiente: a) gastroenteritis aguda b) gastritis c) parasitosis intestinal d) hepatitis viral e) neumita lobar aguda 75.- el agente causal de esta entidad patológica es: a) virus de la hepatitis A b) salmonella tiphy c) áscaris lumbricoides d) neumococo e) helicobacter pilory 76.- en un estudio radiográfico usted espera encontrar: a) condensación basal b) dilación de asas intestinales c) hepatomegalia d) cámara gástrica distendida e) edema interasa 77.- epidemiologicamente predomina en: a) lactantes b) preescolares c) escolares d) adultos e) ancianos 78.- una complicación de esta patología es: a) obstrucción intestinal b) sangrado de tubo digestivo c) restricción de la ventilación pulmonar d) acidosis metabólica e) insuficiencia hepática Caso 23.En la consulta de urgencias se atiende a una mujer de 50 años de edad que se presenta con desorientación evidentes, cambios de conducta y agresiva durante el interrogatorio es analfabeta de condiciones socioecomicas deficientes, ingiere de 1 a 2 litros de pulque al día,

12

palidez de tegumentos, ictericia conjuntival HB de 7.8, Hematocrito de 31%, leucocitos de 6.300, plaquetas de 73.500, TP 1 9 segundos, TPT de 60 seg. 79.- son datos clínicos para fundamentar el diagnostico pero fueron omitidos: a) signo de la rueda dentada, hiperreflexia y temblor b) signo plantar extensor, eritema palmar y desnutrición c) hiporreflexia, asterixis, púrpura d) telagiectasias, ascitis, asterixis e) hematomas en varios lados, alopecia, deshidratación 80.- cual de las siguientes causas no es precipitante del cuadro: a) diarrea b) anemia c) hiperkalemia d) trasgresión dietética e) hipervolemia 81.- de los siguientes medicamentos ¿Cuál puede precipitar a este cuadro? a) propanolol b) furosemida c) lactolosa d) neomicina e) bromocriptina Caso 24.Paciente masculino de 43 años de edad, administrador de empresas , con tabaquismo positivo de 40 cigarrillos diarios, acude por dolor epigastrio intenso de 4 años de evolución que cede con la ingestión de alimentos y que rearase 40 minutos después. No ha recibido tratamiento previo a la exploración física se encuentra normal: 82.- el diagnostico de mayor probabilidad es: a) gastritis crónica b) gastritis aguda c) ulcera duodenal d) hernia hiatal e) ulcera peptica 83.- epidemiologicamente el padecimiento diagnosticado se caracteriza por ser: a) más comun en mujeres b) raro en la población general c) comun en menores de 60 años d) más frecuente en grupo sanguineo B e) comun en neumopatas 84.- para conformar el diagnostico solicita: a) esofagograma b) serie esófago gastrodoudenal c) gástrica serica d) endoscopia e) manometria 85.- el tratamiento indicado en este paciente es: a) bloqueador H1 b) bloqueador H2 c) inhibidores de la bomba de protones d) antiácidos e) procineticos 86.- la complicación mas común del padecimiento que puede presentar el paciente es: a) hemorragia b) perforación c) penetración d) obstrucción del vaciamiento gástrico

13

e) infección Caso 25.hombre de 56 años de edad que acude por dolor epigastrio de tres días de evolución de intensidad moderada continua punzante e irradiado a hipocondrio derecho, acompañado de hipertermia de 39º C, disnea de decúbito y tos seca intermitente que exacerba el dolor. El paciente se encuentra en mal estado de higiene esta pálido delgado con pulso de 100 por minuto, hipomotilidad, murmullo vesicular disminuido en base pulmonar derecha y con el hígado palpable a 2 centímetros por abajo del borde costal en la línea medio clavicular hay dolor a la digitopresion del borde hepático. 87.- el antecedente mas frecuente que usted esperaria encontrar en este paciente es: a) ingestión de pulque b) hacinamiento c) dieta deficiente en calidad y cantidad d) medio socioecomico bajo e) hepatopatia 88.- la radiografía de tórax en este caso mostrara probablemente una de las siguientes alteraciones: a) elevación del hemidiafragma derecho b) paquipleuritis c) infiltrado basal derecho d) derrame pleural derecho e) condensación basal derecha 89.- uno de los siguientes procedimientos es el más útil para el diagnostico: a) amiba en fresco b) radiografía simple de abdomen c) seroamiba d) ultrasonografia hepática e) prueba de función hepática 90.- el tratamiento de elección será: a) diyodoquinoleina b) cloroquina- metronidazol c) emetina-tinidazol d) quinfamida-metronidazol e) metronidazol o dehidroemetina 91.- en caso de respuesta desfavorable usted indica: a) cambiar esquema antibiótico b) punción y drenaje c) laparotomía d) laparotomía y tratamiento medico e) punción y drenaje mas tratamiento medico Caso 26.el 5 de mayo de 1996, acude una mujer de 18 años de edad con antecedentes ginecoobstetricos G: 1, P:0, fecha de ultima regla 22-09-1995, amenaza de aborto a los 8 semanas de gestación e infección de vías urinarias en la semana diez de embarazo, tratada con ampicilina por 8 días. Refiere salida de liquido transvaginal claro desde hace 5 horas posteriormente se agrava dolor abdominal, el cual persiste hasta el momento actual. Tension arterial de 110/60 mmHg, frecuencia cardiaca de 80 por minuto, temperatura de 36.5º C, fondo uterina de 29cm por arriba de la sínfisis del pubis, dorso a la derecha, frecuencia cardiaca fetal de 144 por minuto, buena actividad fetal, actividad uterina con 3 contracciones en l0 minutos y 6 duración de 30 a 40 segundos. Cerviz central con borramiento del 60% y dilatación de 3-4cm la presentación es cefálica, el signo de tarner positivo y las membranas planas. 92.- si la gestación termina en este momento de acuerdo con la edad estacional hablamos de: a) parto inmaduro

14

b) aborto del segundo trimestre c) parto prematuro d) parto de término e) parto distócico 93.- en este caso, el trabajo de parto efectivo se presentara cuando: a) la frecuencia cardiaca fetal sea de 144 por minuto y borramiento cervical b) las contracciones en 10 minutos sean tres sin triple gradiente descendente c) las contracciones en 10 minutos sean 3 con duración de 40 segundos d) la frecuencia cardiaca fetal sea de 144 por minuto y las contracciones en 10 minutos sean tres. e) el borramiento cervical. 94.- ocurrida la ruptura prematura de membranas, la madre y el producto deben considerarse potencialmente infectadas, una vez transcurridas más de: a) 2 horas b) 3 horas c) 4 horas d) 6 horas e) 8 horas 95.- la complicación mas frecuente de ruptura prematura de membranas es: a) endometritis b) corioamnioitis c) sepsis neonatal d) perforación e) schok séptico 96.- es una característica del líquido amniótico neonatal en el embarazo a término: a) concentración alta b) cantidad mayor de 2000 mililitros c) menos del 10% de células naranja d) densidad de 1.008 y pH de 7.2 e) prueba negativa a sustancia tensoactiva. Caso 27.Acude a consulta una paciente mujer de 25 años de edad, sin antecedentes de importancia a la que se le solicita prueba inmunológica de embarazo la cual se reporta positiva. Después de interrogar la historia clínica usted decide abrir el expediente de la paciente. 97.- cuales son los estudios de laboratorio de primera opción que se solicitaría a una paciente: a) BH, glucosas sanguíneas, grupo sanguíneo, Rh y VDRL b) BH, EGO, glucosa sanguínea, grupo sanguíneo, Rh y VDRL c) BH, grupo sanguíneo, Rh, TP, USG y VBDRL d) acido úrico, urea, creatinina, glucosa sanguínea, grupo sanguíneo, Rh. VDRL. e) EGO, colesterol total, triglicéridos, BH, glucosa sanguínea, grupo sanguíneo, Rh, VDRL. 98.- la biometría hematica reporta 10.5g de hemoglobina y de acuerdo con este dato ¿Qué tipo de anemia presenta la paciente?: a) hipocromica b) normocitica c) microcitica d) hipercromica e) macrocitica 99.- se considera que el metabolismo del hierro estará aumentado en este paciente movilizando sus reservas y aumentara su absorción. ¿Cuál dosis diaria recomendaría de este mineral? a) 40-50 mg b) 60-80 mg c) 100-120 mg d) 150-300 mg

15

e) 400-500 mg 100.- con respecto al gasto cardiaco se considera que los cambios adaptativos que sufrirá esta paciente son: a) aumento del 10-19% b) aumento del 20 al 29% c) aumento del 30 al 49% d) aumento del 40 al 49% e) aumento del 50 al 59% 101.- los cambios de filtración glomerular y el flujo plasmático serán: a) aumento del 10 al 29% b) aumento del 30 al 49% c) aumento del 50 al 69% d) aumento del 70 al 89% e) aumento del 99 al 100%. Caso 28.Primigesta de 18 años de edad con embarazo de término, sin trabajo de parto con antecedentes de edema y cefalalgia. Inicia hace 1 hora con dolor en abdomen bajo a la exploración física se encuentra con frecuencia cardiaca fetal de 140 por minuto, tension arterial de 60/40, abdomen doloroso no se escucha foco fetal.

102.- el diagnostico más probable es: a) desprendimiento de placenta b) sufrimiento fetal c) tromboembolia pulmonar d) perforación uterina por mola hidatiforme e) síndrome de hipotensión supina 103.- los antecedentes se asocian comúnmente con: a) preeclamsia-eclampsia b) placenta previa c) insuficiencia venosa profunda d) enfermedad trofoblastica e) comprensión de la vena cava inferior 104.- la complicación inmediata de la madre será: a) hemorragia y choque b) sepsis e infección c) atonia uterina d) inversión uterina e) perforación uterina 105.- esta complicación es mejor tratada con: a) fibrinogeno b) infusión sanguínea c) histerectomía d) manitol e) estimularon uterina con oxitocina Caso 29.Se presenta a consulta paciente de 18 años de edad con vida sexual activa, no utiliza método de planificación familiar y presenta retraso menstrual de 9 semanas. 106.- de acuerdo a su fecha de última regla el estudio más confiable para el diagnostico de embarazo es: a) radiografía de abdomen b) prueba de fridman

16

c) tomografía de abdomen d) prueba inmunológica e) perfil foliculoestimulante, luteinizante, estrógenos y progesterona 107.- el signo que es posible detectar en la exploración física es: a) mac burney b) gestosico c) hegar d) motgomery e) chadwick 108.- para este tiempo de amenorrea ¿Qué tamaño se espera que tenga el fondo uterino? a) 4 centimetros b) 8 centimetros c) 12 centimetros d) 16 centimetros e) no se detecta 109.- se espera que el próximo mes la paciente reporte ganancia ponderal de: a) 250 g. b) 500g c) 750 g d) 1 000 g e) 1 250 g

110.- se considera que presenta hipertensión arterial durante su embarazo, si el aumento de la presión arterial diastolita es de: a) 5 mm Hg b) 10 mm Hg c) 15 Mm. Hg d) 20 mm Hg e) 25 mm Hg Caso 30.se presenta a consulta de medicina familiar, una paciente primigesta de 25 años de edad que cursa con embarazo de 39 semanas de gestación y desde hace 6 hrs. tiene contracciones uterinas con frecuencia de 3 en 10 minutos y duración aproximada de 40 segundos, además menciona salida de moco sanguinolento a trabes de la vagina. 111.- la conducta que se debe asumir es: a) tomar los signos vitales y solicitar exámenes de laboratorio b) exploración ginecológica y valora su envió a urgencias tocoquirurgicas c) envió inmediato a urgencias tocoquirurgicas d) descartar ruptura prematura de membranas e) toma de presión arterial, medición del fondo uterino y au scultación de foco fetal 112.- en la exploración ginecologota se encuentra cerviz central con dilatación de 2 cm, borramiento del 50%, membranas integras por lo que se considera que el trabajo de parto esta en: a) pródromos b) 1er periodo c) 2do periodo d) 3er periodo e) expulsivo 113.- tomando en cuenta la duración de este periodo en las pacientes primigestas usted decide: a) enviar a urgencias tocoquirurgicas en cuanto tenga los resultados de laboratorio b) enviar a la unidad tocoquirurgica con calma ya que este periodo dura algunas horas c) observarla cuatro a seis horas y luego la canaliza a la unidad tocoquirurgica d) citada en 24 horas para nueva evaluación

17

e) mandarla a su domicilio para que se bañe y recoja sus objetos personales porque tiene tiempo de sobra Después, la paciente es valorada y se encuentra con borramiento del cervix en un 100%, dilatación de 10 cm y membranas integras, por lo que se procede a romper las membranas y se canaliza a sala de expulsión. 114.- este periodo se considera normal si su duración no excede de: a) 10 minutos b) 30 minutos c) 60 minutos d) 90 minutos e) 120 minutos 115.- después del parto se presenta el alumbramiento sin contratiempos: la secuencia del alumbramiento es: a) desprendimiento, descenso, expulsión b) descenso, desprendimiento, expulsión c) infiltración, expulsión, desprendimiento d) descenso y desprendimiento e) desprendimiento y expulsión

Caso 31.Se trata de una mujer de 30 años de edad con el antecedente de presentar irregularidades menstruales desde la pubertad y posteriormente amenorrea y esterilidad. A la exploración física se aprecia obesidad e hirsutismo, las pruebas de laboratorio revela niveles elevados de LH, testosterona y estrógenos, con FSH normal. El ecosonograma muestra ovarios crecidos. 116.- el diagnostico más probable en esta paciente será: a) hiperplasia suprarrenal congénita b) tumor esofágico funcional c) hiperandrogenismo d) ovarios poliquisticos e) teratoma ovárico 117.- el tratamiento mas adecuado para esta paciente será: a) clomifeno b) progestagenos c) estrógenos conjugados d) reseccion de ovarios e) clomifenos mas estrógenos 118.- si la paciente quiere ser fértil cual será el tratamiento adecuado: a) clomifeno b) progestagenos c) estrógenos conjugados d) reseccion de ovarios e) clomifenos mas estrógenos Caso 32.Ingresa a sala de labor primigesta de 26 años de edad que cursaron 39 semanas de amenorrea confiable, sin vigilancia prenatal. Presenta dolor intermitente tipo cólico en hipogastrio acompañado de perdida transvaginal de líquido amniótico desde hace 30 minutos. En la exploración física se escucha foco cardiaco fetal con 150 latidos por minuto de 3 a 5 contracciones en 10 minutos con 30 segundos de duración: el cuello uterino completamente

18

borrado con 6 cm de dilatación, palpándose a través del miembro de las espinas ciáticas. Se determina que los estrechos superiores, medio e inferior son amplios. Se detecta la presencia de meconio. 119.- el diagnostico clínico obstétrico es embarazo de término: a) en segundo periodo de trabajo de parto, feto en presentación pélvica completa, ruptura prematura de membranas. Sufrimiento fetal agudo. b) en segundo periodo de trabajo de parto, feto en presentación pélvica incompleta en sacro percha anterior. Ruptura precoz de membranas. c) en trabajo de parto, en el periodo de dilatación y borramiento. Producto en presentación pélvica. Sufrimiento fetal agudo. d) en primer periodo de trabajo de parto. Producto en presentación pélvica franca de nalgas en sacar derecha anterior. Ruptura precoz de membranas e) en trabajo de parto, en el periodo de dilatación y borramiento. Ruptura prematura de membranas, sufrimiento fetal. 120.- de acuerdo con los criterios de Hodge la presentación fetal se encuentra en el: a9 primer plano pélvico b) segundo plano pélvico c) tercer plano pélvico d) estrecho medio de la pelvis e) estrecho superior de la pelvis

121.- el diámetro anteroposterior o diámetro conjugado verdadero tiene como referencias: a) coxal y pelvis b) promontorio y pubis c) coxal y promontorio d) isquion y cóccix e) espina ciática y espina ciática 122.- el Angulo obtuso hacia delante, formado entre el cuerpo uterino y el cerviz se denomina: a) posición b) versión c) flexión d) punto toconomico e) presentación 123.- la conducta a seguir de acuerdo con el diagnostico es: a) analgesia obstétrica con bloqueo peridural. Interrupción inmediata del embarazo por operación cesárea b) anestesia general, realización de maniobras de versión interna y esperara parto vaginal. c) impregnar con antimicrobianos e interrupción inmediata del embarazo d) anestesia general e interrupción intermitente del embarazo e) analgesia obstétrica, realización de maniobras de versión interna y esperara parto vaginal con aplicación de fórceps. Caso 33.Se trata de una mujer de 66 años de edad que acude a consulta por sangrado transvaginal. Es obesa y tiene antecedentes de hipertensión y diabetes. El examen pélvico es normal, excepto por una cantidad moderadaza de sangre proveniente del útero. 124.- el diagnostico más probable es: a) desprendimiento hormonal b) mioma submucoso c) adenocarcinoma de endometrio

19

d) sarcoma uterino e) carcinoma del cercix 125.- esta lesión comúnmente: a) es precedida por cervicitis crónica b) esta relacionada con el virus del papiloma humano c) es precedida por hiperplasia adenomatosa d) se ve generalmente en mujeres con menopausia temprana e) sigue a una ooferectomia bilateral 126.- el tratamiento aceptado de esta entidad incluye: a) únicamente cirugía b) cirugía y hormonas c) hormonas y radioterapia d) cirugía y radioterapia e) únicamente radioterapia Caso 34.Se recibe en la consulta de control de crecimiento y desarrollo a un lactante menor de edad, que al nacimiento peso 3 100 Kg. La talla de 50 cm y el periodo cefálico de 35 cm. 127.- de acuerdo a la edad actual las medidas que debe tener son: a) peso de 6 Kg, talla 60 cm, PC 40 cm b) peso de 6 400 Kg, talla 62 cm, PC 42 cm c) peso de 7 100 Kg, talla de 66 cm, PC 44 cm d) peso de 6 500 Kg, talla 64 cm, PC 46 cm e) peso de 6 200 Kg, talla 58cm, PC 48 cm

128.- en la exploración física se aprecia en boca, brote de dientes, incisivos inferiores, la secuencia normal de la erupción dental es: a) incisivos centrales inferiores, incisivos centrales superiores, incisivos laterales superiores, incisivos laterales inferiores b) incisivos centrales superiores, incisivos cenarles inferiores, incisivos laterales inferiores, incisivos laterales superiores c) incisivos laterales superiores, incisivos centrales inferiores, incisivos centrales superiores, incisivos laterales inferiores d) incisivos laterales inferiores, incisivos centrales inferiores, incisivos centrales superiores, incisivos laterales superiores e) incisivos inferiores centrales, incisivos laterales inferiores, incisivos centrales superiores, incisivos laterales superiores 129.- se le coloca en decúbito prono y se aprecia que es capaz de rodar sobre su propio eje. Esta conducta indica que el desarrollo neurológico del niño es: a) adelantada en su desarrollo neurológico b) no tiene significado para su desarrollo c) normal para su edad d) retraso en su desarrollo psicomotris e) no acorde para su edad 130.- este lactante es capaz de reírse y de emitir sonidos vocales repetidos. La sonrisa social aparece a la edad de: a) 8 semanas b) 10 semanas c) 12 semanas d) 16 semanas e) 20 semanas 131.- en el desarrollo de esta paciente es normal encontrar que: a) pronuncia sus primeras silabas b) sea capaz de iniciar la deambulacion c) ponga en posición sus dedos índice y pulgar

20

d) pueda soltar un objeto si se hace la petición e) muestre preferencia por la persona que lo prodiga 132.- su requerimiento diario de carbohidratos por kilo de peso es de: a) 5- 7 g b) 10-15 g c) 17- 27 g d) 30- 40 g e) 45- 70 g Caso 35.Después del trabajo de parto normal, nace producto del sexo masculino, con llanto espontáneo al que se le realizan maniobras de rutina para su cuidado. El peso del niño fue de 3 250 Kg. La talla de 51 cm y el perímetro cefálico de 35 cm. 133.- con los datos anteriores el diagnostico es recién nacido: a) hipertrofico b) peso bajo c) eutrófico d) distrófico e) peso alto para la edad gestacional 134.- la calificación de APGAR al 1º y 5º minutos es de 9-10, por lo que se le considera: a) delicado y pronostico reservado b) diagnostico y pronostico favorable c) delicado y pronostico favorable d) normal y pronostico reservado e) delicado y pronostico normal

135.- la alimentación se presenta: a) a las 3 horas con formula materna previo lavado gástrico b) a la hora con solución glucosaza c) inmediatamente con solución glucosaza d) a las 3 horas con formula maternizada e) inmediatamente con leche materna A las 2 horas de vida extrauterina su frecuencia cardiaca es de un promedio de 130 latidos por minuto, ala exploración física se localiza el choque del apunta del corazón fuera de la línea media clavicular entre el 3º y 4º espacio intercostal. 136.- el diagnostico correspondiente es: a) cardiopatía congénita b) taquicardia c) insuficiencia cardiaca d) se considera normal e) cardiomegalia A las 24 horas de vida extrauterina, el paciente presenta respiración abdominal con frecuencia de 50 por minuto y en la auscultación de campos pulmonares se identifican estertores muy leves. 137.- usted considera que se trata de: a) insuficiencia respiratoria aguda b) síndrome de membrana hialina c) neumonía del neonato d) se considera normal e) aspiración de meconio Caso 36.-

21

En el cunero fisiológico se encuentra un recién nacido de 24 horas de vida extrauterina sin antecedentes de importancia se le realiza estudio neurológico presentando los siguientes reflejos: moro, tronco del cuello, prension, succión y cremasteriano 138.- el diagnostico de esta paciente es desarrollo neurológico: a) incompleto b) acelerado c) inadecuado d) anormal e) normal 139.- reporta que presento dos evacuaciones meconiales. La medida que debe seguirse al respecto es: a) ninguna se considera normal b) lavado gástrico c) suspender la vía oral d) realizar Rx simple de abdomen e) vigilar distensión abdominal 140.- en el examen de orina se reportan cetonas, cilindros, leucocitos y eritrocitos menos de 10 por campo lo que sugiere: a) riñón polquistico b) malformaciones del sistema pericalicial c) urosepsis d) orina de características normales e) riñón en herradura 141.- en el recién nacido hay diferencia de glucoroniltranferasa ¿Cuál de los siguientes antibióticos esta contraindicado a esta edad debido a que se excreta en la orina como glucoronido en un 90%? a) cloramfenicol b) tetraciclina c) eritromicina d) cloropromacina e) penicilina G Caso 37 Un niño de 4 años de edad presenta hematuria indolora. Los padres no recuerdan enfermedad previa, traumatismo o molestias para orinar. El niño refiere solamente cefalea. A la exploración física: tensión arterial de 132/84 mmHg, faringe hiperemica y discreta edema palpebral. Por laboratorio BH Hb 10.5 g/100ml, leucocitos 10.200/mm3, EGO Densidad1020, leucocitos 1.2 por campo, eritrocitos abundante proteínas + +. Descenso discreto de albúmina serica, nitrógeno ureico y velocidad de eritrosedimentacion elevados 142.-El diagnostico mas probable es: A) UROSEPSIS B) LITIASIS ureteral c) glomerulonefritis crónica d) síndrome nefrotico e) piolenefritis aguda

143.-¿cual factor es indicativo de mal pronóstico? A) inicio del cuadro entre 2-4 años b) hipertensión arterial leve c) no responde al tratamiento d) hematuria o retención de nitrógeno no proteico e) desaparición del edema y albuminuria persistente Caso 38 Un paciente femenino de 22 años de edad ingreso al hospital refiriendo que hace 2 meses inicio de ataque al estado general, adinamia, hiporexia, perdida de 6 KG. Lipotimias, mareos, agregándose posteriormente edema palpebral, disnea de medianos esfuerzos, nauseas,

22

vómitos y oliguria. Acudió a su hospital para diálisis peritoneal con catéter rígido. A la exploración física presión arterial 130/110 mmHg, palidez generalizada. Hemoglobina 10.2 g %, hematocrito 31 %, creatinina de 16.2% urea de 171 mg % k=6.5 mEq/l. acido úrico de 9 mg/dL. Nefrectomía con riñones disminuidos de tamaño. Se inicio diálisis peritoneal con catéter rígido, 6 días después se le coloco blando tipo TencKoff. A los 15 días ingreso dolor abdominal, vomito, afebril con rebote positivo y dolor a la palpación profunda en falco izquierdo y mesogastrio, perístasis presente. Se continúo con diálisis peritoneal intermitente. Alos 3 meses se detecto hemoglobina de 5.6g% hematocrito 22% y se transfundió. 144.- cual es la indicación de diálisis peritoneal: a) K > 5 mEq/l b) K > 3mEq/l c) HCO3 > 45 d) BUN > 16 e) Na > 145 mEq/l 145.- en esta paciente el diagnostico que se realiza con base en los datos clínicos es: a) insuficiencia renal aguda b) glomerulonefritis rápidamente progresiva c) insuficiencia renal crónica d) nefritis intersticial e) síndrome nefrotico 146.- en esta paciente, el aumento de la concentración plasmática de urea y creatinina se debe a: a) disminución en la reabsorción tubular de fósforo y potasio b) disminución de la filtración glomerular c) aumento en su síntesis d) disminución de la diuresis por hipovolemia e) disminución del catabolismo renal en estas sustancias

147.- la hipertensión arterial en este caso es primordialmente secundaria a: a) isquemia renal b) aumento del gasto cardiaco c) aumento del a aldosterona d) aumento del sistema renina angiotensina e) aumento del factor natriuretico atrial 148.- la indicación urgente de la diálisis en esta paciente fue por: a) hioperuricemia b) insuficiencia cardiaca c) retención de sodio d) hiperkalemia e) hipercreatinemia 149.- una indicación dietética para el manejo de la uremia es: a) disminuir la ingesta de sodio b) limitar lo indispensable la ingesta de proteínas c) disminuir la ingesta de sodio d) disminuir la ingesta de grasas saturadas e) disminuir la ingesta de potasio Caso 39: Paciente femenina de 71 años de edad con diabetes mellitus de 25 años de evolución tratada con glibenclamida y ahora con autocontrol. Se sabe hipertensa desde hace 2 años con tratamiento no especificado. Además, desde hace una semanas presenta poliuria con nicturia de 5 y urgencia de micción Exploración física paciente obesa con anasarca, alerta con fascies de angustia, palidez de tegumento, ojo derecho con catarata e izquierdo con exudados algodonosos y perdida del patrón vascular, mal hidratada, tórax con hipomotilidad. Ruidos cardio-respiratorios rítmicos

23

con desdoblamiento del 2º ruido. Abdomen con ascitis, signo de la ola positivo, dolor a la palpación de ambos flancos. 150.- con los datos presentados, usted puede interpretar el diagnostico siguiente: a) Glomerulonefritis aguda b) Glomerulonefritis cronica c) Pielonefritis d) Nefritis intersticial e) insuficiencia renal 151.- en los estudios de laboratorio usted encontrara: a) Hb normal, alcalosis metabólica, creatinina ligeramente alta, proteínas normales. b) Hb baja, hipoproteinemia, creatinina elevada, acidosis metabólica c) Hb normal, alcalosis metabólica, creatinina elevada, hipoproteinemia d) Hb baja, acidosis metabólica, creatinina normal, hiperkalemia e) Hb baja, acidosis metabólica, creatinina normal, hipoproteinemia 152.- la acidosis que presenta la paciente se debe a: a) hipoproteinemia b) hipertensión portal c) efecto osmotico d) secundario a daño renal e) insuficiencia cardiaca derecha 153.- el tratamiento de elección es: a) dieta baja en proteínas b) antihipertensivos c) diálisis peritoneal d) nefrectomía e) corticoesteroides 154.- la complicación que se presenta mas frecuentemente con el tratamiento indicado es: a) peritonitis b) hipocalcemia c) choque séptico d) cushing e) uremia Caso 40 Un paciente masculino de 65 años de edad es llevado al medico por presentar depresión. Al interrogatorio refiere perdida de la memoria reciente en forma progresiva e insidiosa. En ocasiones ha tenido problemas con sus familiares debido a su juicio alterado. Quejándose además de debilidad. Ha llegado a pensar que se convierta en su padre. Se le realizan valoraciones neurológicas y psiquiatrika y se le toman estudios de tomografía axial computarizada (TAC). Con lo cual se determina el diagnostico de enfermedad de alzheimer. 155.- la edad en que se diagnostica más rápidamente la enfermedad de Alzhimer es a los: a) 80 años b) 70 años c) 60 años d) 55 años e) 50 años 156.- el diagnostico de la enfermedad de Alzheimer se establece por la presencia de: a) depresión b) demencia c) ataxia d) alucinaciones visuales e) estado maniaco 157.- los hallazgos del TAC son: a) borramiento de surcos corticales

24

b) zonas de isquemia cerebral c) infartos lacunares d) desmielinizacion cerebral difusa e) atrofia y dilatación ventricular 158.- las alteraciones específicas de la enfermedad de Alzheimer son: a) afección de la vía extrapiramidal b) cuerpos en inclusión c) degeneración de la sustancia negra d) atrofia del parénquima cerebeloso e) placas seniles, degeneración neurofibrilar 159.- en el paciente presentaba trastorno motor caracterizado por alteraciones del habla. Se debe valorar aspectos como el ritmo, la entonación, prosodia y la melodía. ¿A cual de los siguientes especialistas lo referiría? a) fonista b) neurólogo c) lingüística d) otoneumologo e) audiologo Caso 41 Paciente femenino de 70 años hipertensa desde hace 15 años actualmente control con nifedipino. Inicia su padecimiento hace 4 horas al presentar subitamente somnolencia, desviación de la comisura labial a la izquierda dificultad para articular palabras, además de palidez generalizada y dificultad para la respiración. En la exploración física se encontró con somnolencia, afasia motora, lesión del VII par craneal, hemiplejía derecha, reflejos osteotendinos abolidos, Babinsky y Chadkon derecho. 160.- con estos datos usted puede integrar el siguiente diagnostico: a) ictus b) hidrocefalia c) encefalopatía hipertensiva d) hemorragia parenquimatosa e) hemorragia cerebelosa 161.- ¿Cuál es el tipo de lesión? a) ipsilateral b) localizada c) multifocal d) intratentorial e9 ganglios básales 162.- la vía afectada es: a) extrapiramidal b) piramidal c) ascendente d) vía propioceptiva e) sistema limbito 163.- la lesión se ubica en: a) frontal parietal derecho b) frontal izquierdo c) fascículo de Buroach d) núcleo basal e) lados de la cisura interhemisferica 164.- el tratamiento de urgencias que usted indicaría de primera intención seria: a) corticoesteroides b) diuréticos c) dipiradamol IV d) antihipertensivos e) soluciones

25

Caso 42 Paciente femenina de 24 años de edad, epiléptica controlada con DFH y carbamacepina es llevada a urgencias por amigos, quienes refieren que hace 10 minutos, la paciente presento convulsión tónico clónica generalizada con perdida del estado de alerta, micción espontánea. A su ingreso persiste contracciones tónico-clónicas generalizadas, inconciente trismos, y mirada fija. Al examen físico la tension arterial de 110/70 mmHg la frecuencia cardiaca de 88 por minuto, la frecuencia respiratoria de 20 por minuto, la temperatura de 36º C. no hay compromiso cardiaco, ventilatorio, ni abdominal, babisky y sucedáneos ausentes. 165.- el manejo terapéutico inicial corresponde a: a) administrar solución salina al 0.9% b) asegurar una función cardiorrespiratoria adecuada c) asegurar la administración de fenobarbital IV d) asegurara de tomar inmediatamente TAC de cráneo e) aplicar medidas antiedema cerebral 166.- la dosis inicial por minuto de diacepam IV es de: a) 0.5 mg b) 1 mg c) 2 mg d) 3 mg e) 5 mg 167.- la dosis IV de DFH en mg/Kg es de: a) 5 a 8 b) 9 a 14 c) 15 a 18 d) 19 a 22 e) 23 a 25 168.- a pesar del manejo anterior, la crisis convulsiva persistente después de 30 minutos, por lo que sugiere administración de: a) lidocaina b) carbacepina c) DFH d) tiopental e) DFH y lidocaina 169.- la resolución del estado epiléptico debe ser en menos de: a) 1 hora b) 4 horas c) 8 horas d) 12 horas e) 1 día Caso 43 Paciente femenina de 40 años de edad que acude al servicio de urgencias , refiere que inician su padecimiento desde hace una semana al presentar parestesias en mejilla derecha y acufeno en oído derecho hasta hace 24 horas en que se presenta retracción de la comisura bucal hacia la izquierda, sialorrea, dificultad para masticar. Se acompaña de perdida de la audición en el oído derecho. Antecedentes de importancia, hipoacusia de 24 horas de evolución. Niega antecedentes de hipoacusia familiar, exposición a ambiente ruidosos y fármacos ototoxicos. Hace una semana cursa con un cuadro gripal, recibió tratamiento con antihistamínicos. A la exploración física: Inquieta, conciente, con asimetría facial a expensas de retracción de comisura hacia la izquierda y ausencia de pliegues frontales, disminución de la hendidura palpebral del lado derecho. Exploración neurológica se encuentra afectados los nervios craneales VII, con falta de movilidad de la musculatura facial y VIII con huecas profundas sensorial unilateral derecha, audición normal en oído izquierdo. El resto de los pares craneales no mostraron alteraciones.

26

170.- los datos mencionados corresponde al diagnostico de: a) síndrome de ransan Hunt b) parálisis facial recurrente c) parálisis de hialob d) parálisis de bell e) parálisis de weber 171.- entre los factores precipitantes de esta etiología se encuentra: a) exposición al frió b) infecciones parenterales c) infecciones respiratorias virales d) traumatismos e) herpes zoster 172.- el agente etiológico en este caso es: a) influenza virus b) haemophilus influenzae c) herpes zoster d) neisseria Meningitiditis e) Escherichia 173.- el tratamiento de elección en este caso es: a) antiinflamatorios no esteroides y antibióticos b) antiinflamatorios esteroides y vitamina B 12 c) antibióticos d) antiinflamatorios no esteroides e) antivirales

Caso 44 Paciente masculino de 17 años de edad es llevado al consultorio por sus padres, argumentando que desde hace 4 meses temerosos, se aislaba, no sala de su casa, se observa con frecuencia al espejo, esa sintomatología cada día era mas notorio pero desde hace un mes se intensifica y presenta soliloquias, sin motivo aparente al rechazo al alimento porque asegura lo quieren envenenar. En la entrevista el paciente acepta tener alucinaciones auditivas y asegura no ser el mismo, sino que hay alguien en su interior que control sus acciones y pensamientos, lo obliga a tomarse ocasionalmente violento. 174.- en el caso anterior podría corresponder a: a) esquizofrenia hebefrenica b) delirio c) esquizofrenia paranoide d) trastorno paranoide de la personalidad+ e) trastorno delirante paranoide 175.- según la respuesta anterior y para cumplir con criterios diagnosticas usted esperaba a: a) la aparición de un episodio maniaco b) la aparición de un episodio depresivo c) la evolucion durante 2 meses d) evolucion durante 6 meses e) la evolucion del padecimiento 175 A. Una vez que la fase agresiva ha pasado usted espera encontrar: a) aislamiento

27

b) deterioro en el aseo e higiene personal c) apetito desmedido e intensa participación social d) retraimiento social e) afectividad embotada 176.- con respecto a la personalidad premorbio en este tipo de paciente, usted pensaría en rasgos de personalidad como: a) suspicaz e introvertida b) exhibiocinista y compulsiva c) extrovertida y obsesiva d) dependiente y agresiva e) labil e independiente Caso 45 ¿Acude a consulta un hombre de 40 año s de edad a quien le diagnostica neurocistercosis? 177.- ¿Qué tratamiento decide usted iniciar? a) prazicuantel b) niclosamida c) quinfamida d) albendazol e) pirantel 178.- suele haber reacción inflamatoria severa en sistema nervioso central cuando los cisticercos: a) están vivos en parénquima cerebral b) están calcificados en parénquima cerebral c) están muertos y en degradación d) afectan ventrículos cerebrales e) están en fase larvaria 179.- antecedente que confirma el diagnostico: a) infección intestinal con Taenia Solium b) infección por Clonorchis Sinesus c) esquistosomiasis d) anemia megaloblastica macrocitica e) linfadenopatia cerebral

Caso 46 Paciente masculino de 50 años de edad que refieren cuadros de neumonías frecuentes desde hace 2 años. Posteriormente se agrego astenia e hiopoadinamia con perdida de 10 kilogramos en el lapso de un año. Tiene el antecedente de que hace seis meses sufrió fractura de clavícula sin que existiera traumatismo. 180.- en este caso el diagnostico más probable es: a) síndrome de inmunodeficiencia adquirida b) mieloma múltiple c) hiperparatiroidismo d) macroglubilenemia de Waldenstrom e) carcinoma metastático de tiroides 181.- Cual es el método que confirmaría el diagnostico? a) ELISA para VIH b) biopsia de medula ósea c) calcio, fosforo, y fosfatasa alcalina d) velocidad de sedimentación globular, inmunoglobulinas sericas e) determinación de niveles sericos de parathormona 182.- las células que sintetizan inmunoglobulinas circulantes son: a) monolitos b) linfocitos T c) linfocitos B

28

d) plasmáticas e) mastocitos 183.- el linfocito B se diferencia en célula plasmática fundamentalmente en: a) hígado b) timo c) medula ósea d) placas de pleyer e) sistema reticuloendotelial 184.- los linfocitos b poseen una inmunoglobulina que determina su diferenciación y especificación en células plasmáticas y se localiza en: a) núcleo b) citoplasma c) aparato de goldi d) membrana citoplasmática e) reticuloendoplastico rugoso 185.- en este caso es común encontrar como hallazgo de laboratorio: a) trombocitopenia b) células plasmáticas atípicas c) hipocalcemia d) aumento de albúmina e) hiperfosfatemia 186.- el tratamiento indicativo es: a) radioterapia b) corticoides c) quimioterapia d) inmunoglobulina e) inmunoestimulantes Caso 47 Un ejecutivo de 50 años de edad sin antecedentes de importancia despierta en la mañana con dolor agudo en el primer ortejo del pie derecho, después de asistir a un hotel el día anterior. A la exploración física los signos vitales se encuentran normales, el dedo estaba caliente, adolorido y el paciente era incapaz de pararse en pie.

187.- ¿Cuál es el diagnostico mas probable? a) pseudogota aguda b) artritis séptica aguda c) dolor de articulación reumática d) artritis gotosa aguda e) síndrome de reiter 188.- ¿Qué anormalidades es más probable en este paciente? a) aumento del acido úrico serico b) disminución del nivel de complemento serico c) aumento en el titulo sérico de antiestreptolisisnas d) artritis lumbosacra bilateral a los rayos equis e) hemocultivo positivo para gonococo 189.- ¿Cuál complicación puede presentarse en este padecimiento? a) endocarditis bacteriana aguda b) glomerulonefritis proliferativa c) endocarditis bacteriana subaguda d) nefrocalcinosis e) nefropatia por acido úrico 190.- ¿Cuál de los siguientes mecanismos esta involucrado en esta artritis? a) reacción de leves

29

b) proceso inflamatorio c) liberación de histamina d) inhibición de la fagocitosis e) presencia de complejos autoinmunes 191.-¿Cuál es el manejo terapéutico más indicativo para aliviar el dolor de este enfermo? a) aspirina b) EDTA c) colchicina d) diclofenaco e) paracetamol Caso 48 Paciente femenina de 68 años de edad con artralgias en articulaciones de carga de falanges interfalangicas distales de mano, acompañadas de rigidez manital de cinco minutos. A la exploración física se encuentra aumento de volumen de articulaciones interfalangicas distales de manos, crepitación y dolor moderado en rodillas. 192.- usted pensaría como0 posibilidad diagnostica en: a) artritis reumatoide b) enfermedad por depósitos de cristales c) enfermedad articular degenerativa d) gonartrosis e) artritis reactiva 193.- el aumento de volumen de las articulaciones interfalangicas corresponde a: a) nódulos de Bouchard b) nódulos de Heberden c) tofos d) nódulos reumatoides e) nódulos de Osler 194.- los estudios de laboratorio que solicitaría para apoyar su diagnostico: a) biometría hematica completa con sedimentación globular b) factor reumatoide, prueba de látex c) acido úrico d) proteína C reactiva e) complemento hemolítico y sedimentación globular

195.- si solicitaría radiografías de manos esperaria encontrar: a) imágenes osteoliticas dístales interfalangicas b) desviación cubital c) desmineralización yuxta-articular de huesos d) disminución del espacio articular en interfalangicas distales e) osteofitos yuxta- articular 196.- el tratamiento que le daría a este paciente es: a) diclofenaco b) esteroides c) fenilbutazona d) metotrexate e) ciclosporina Caso 49 paciente masculino de 45 años de edad con diabetes mellitus tipo I desde los 15 años de edad, por lo que desde entonces recibe tratamiento con insulina NPH, siendo la dosis actual DE 55u CADA 24 horas. Ingresa al servicio de urgencias procedente de la vía pública y refiere al personal paramedico que se encontraba desorientado y con conducta anormal desde que hizo sus rutinas habituales de ejercicio matutino. Sufre perdida del conocimiento durante el traslado.

30

Al examen físico el paciente se encuentra sudoroso, hipotermico, con aumento d la frecuencia cardiaca y sin respuesta a los estímulos habituales. La química sanguínea reporta glucosa de 40mg/dl y creatinina de 2.5 mg/dl y el examen general de orina glucosa negativa, cuerpos cetonicos negativos, albúmina ++ y cuadros granulosos. 197.- ¿Cuál es el diagnostico de admisión? a) hipoglucemia por utilización de hipoglucemiantes orales b) descontrol cetoacidosico severo de la diabetes c) hipoglucemia por bajo aporte de glucosa en la dieta d) insuficiencia suprarrenal crónica de inicio e) hipoglucemia por aumento de la vida media de la insulina 198.- el agente etiológico corresponde a: a) insuficiencia suprarrenal b) insuficiencia renal c) deficiencia de glucogon d) insuficiencia de células delta e)insuficiencia de células claras 199.- el trastorno fisiológico específico de este padecimiento se debe a: a) liberación excesiva de glucogeno hepático b) insuficiente liberación de glucocorticoides c) excesiva liberación de ácidos grasos d) disminución de la exposición de la insulina e) deficiente captación de glucosa intracelular 200.- se confirma la causa del padecimiento mediante: a) pruebas de función hepática b) prueba de depuración de creatinina c) pruebas de función suprarrenal d) prueba de tolerancia a la glucosa e) toma de biopsia hepática 201.- esta medida terapéutica para este padecimiento es: a) esteroides suprarrenales b) solución glucosada al 5% c) solución salina fisiológica d) solución glucosas al 50% e) solución glucosada al 10%

Caso 50 Paciente femenina de 46 años de edad obesa con predominio en parte superior del cuerpo. Tiene como antecedentes heredofamiliares el que su madre es diabética, acude por un padecimiento de dos meses de evolución, con mareos ocasionales, polifagia, cansancio, leucorrea, prurito vulvar, dolores musculares somnolencia. A la exploración física encontramos tension arterial de 135/90 mmHg, peso de 87 Kg, talla de 1.60 cm., exploración cardiopulmonar normal, abdomen blando no doloroso con peristalsis normal y eritema en áreas intertriginosas. 202.- con los datos anteriores, su sospecha diagnostica es de: a) hipotiroidismo b) hipertensión arterial leve c) síndrome de cushing d) diabetes mellitus e) enfermedad de Graves Basedow 203.- en estos casos, la confirmacion del diagnostico se obtiene inmediatamente : a) perfil tiroideo b) glucemia en ayuno c) angiografía renal selectiva

31

d) captación de yodo e) dosificación de 17 hidroxicorticoesterodes en orina 204.- corroborando el diagnostico, usted inicia tratamiento con: a) prednisona b) propanolol: c) tiroxina d) dieta de reducción e) propituiluracilo 205.- una complicación frecuente que se presenta en forma aguda en este padecimiento es: a) tormenta tiroidea b) coma hiperosmolar c) coma por mixedema d) crisis adisionana e) encefalopatía hipertensiva 206.- otra complicación que se presenta en la historia natural de esta enfermedad a largo plazo es: a) esclerodermia b) atrofia suprarrenal c) cáncer tiroideo d) exoftalmos maligno e) catarata Caso 51 Una mujer de 30 años de edad es vista a causa de episodios de cefalea frecuentes a la exploración física TAS de 450/100 mmHg, el examen de fondo de ojo fue normal. Examen de laboratorio nitrógeno ureico 15 mg%, creatinina serica 0.5mg%, Na serico 145 mEq/l, K 1.9 mEq/l, CO 35 mmol/Hg 207.- el diagnostico más probable es: a) hiperparatiroidismo b) coartación aortica c) estenosis de arteria renal d) hipertensión idiopatica e) aldosteronismo primario 208.- el diagnostico se apoya con: a) soplo parasistolico en la punta b) sodio serico c) telerradiografía de tórax d) hipokalemia e) edema de la cabeza y cuello 209.- son datos a favor de la enfermedad: a) hipertensión, nicturia, estreñimiento b) pulsos alterados c) sodio y cloruro serico bajos, vello axilar escaso dolor abdominal y fiebre d) potasio serico alto, deshidratación, incremento en la pigmentación de la piel y nitrogeno ureico sanguíneo alto e) hipersecreción de aldosterona, retención de sodio y excreción de potasio, aumento de uremia y de la tension arterial. 210.- la terapéutica inicial en este caso es: a) antagonistas de calcio b) bloqueadores alfa adrenergicos c) inhibidores de la ECA d) disminución de peso e) espirinolactona 211.- esta enfermedad puede deberse a: a) alteración primaria de la corteza adrenal b) cortisol deficiente

32

c) destrucción de las cortezas suprarrenales d) hipertensión paroxística o sostenida e) hipercortisolismo Caso 52 un paciente masculino de 60 años de edad acude a la consulta refiriendo ser portador de diabetes mellitus, controlado con glibenclamida 1X3 e hipertensión arterial sistémica controlado con captopril 1X3 , ambas alteraciones tiene 8 años de evolución desde que presento hace una semana ha notado fatiga, edema progresivo de miembros inferiores, ortopnea, palpitaciones, disnea de medianos esfuerzos, disnea paroxística nocturna. A la exploración física tensión arterial 150/100 mmHg, frecuencia cardiaca 112 por minuto, frecuencia respiratoria 24 por minuto, temperatura de 36.2º C, dextroxtix de 86 mg, inquieto con anasarca con plétora yugular, ruidos cardiorrespiratorios sin agregados, campos pulmonares con hipoventilacion basal bilateral, hepatomegalia, edema de miembros inferiores +++. 212.- con los datos anteriores el diagnostico presuntivo es: a) insuficiencia hipertensiva b) cor pulmonar crónico c) insuficiencia cardiaca congestiva venosa d) EPOC descompensada e) enfermedad vascular pulmonar 213.- dentro de las causas precipitantes de esta entidad se encuentra: a) incremento en la ingesta de sal b) obesidad, hipertensión arterial sistémica y diabetes mellitus descompensada c) diabetes mellitus descompensada, historia familiar de insuficiencia vascular d) diabetes mellitus e hipertensión arterial sistémica con insuficiencia renal e) fiebre e hipercoaguabilidad 214.- en la evaluación inicial del paciente, además de la biometría hematica ¿Qué estudios de laboratorio y gabinete solicitaría? a) química sanguínea, pruebas de funcionamiento hepático, Rx de tórax b) química sanguínea, electrolitos sericos, Rx de tórax, electrocardiograma c) electrolitos sericos, creatinina en orina de 24 hrs., rX de tórax d) química sanguínea, transaminasa glutamica oxalacetica, lipoproteinas de alta densidad, Rx de tórax 215.- el tratamiento debe ocuparse de: a) reducir la poscarga, mejorar la función miocárdica b) reducir la precarga y aumentar la poscarga c) aumentar la poscarga y mejorar la función miocárdica d) mejorar únicamente la función miocárdica e) control adecuado de la glucemia y la tensión arterial 216.- la prevención estará enfocada a: a) modificar los factores de riesgo y cumplir con el tratamiento medico b) eliminar estados de stress y ansiedad c) controlar solamente la diabetes mellitus e hipertensión arterial sistémica d) eliminar las causas precipitantes e) monitorización especializada de la función miocárdica Caso 53 Paciente femenina de 35 años de edad que acude a consulta por fatiga, insomnio, perdida de peso, temblor distal. Tiene la piel fina y húmeda, intolerancia al calor. Se nota ligera protrusion ocular y el volumen del cuello ha aumentado en la base del mismo. Tiene palpitaciones, disnea de esfuerzo, así como diarreas frecuentes y edema ligero de piernas. 217.- el diagnostico más probable es: a) hipotiroidismo c) hipertiroidismo c) síndrome de cushing d) neurosis de ansiedad

33

e) ca tiroideo 218.- este padecimiento es mas frecuente en grupos de : a) mujeres b) hombres c) niñas d) niños e) ancianos 219.- el cuadro es compactible con: a) enfermedad de hashimoto b) tiroides de riedel c) carcinoma papilar de tiroides d) enfermedad de Graves e) mixedema 220.- ¿Qué células triyodotironina? a) claras b) principales c) foliculares d) de transición e) parafoliculares

del

parénquima

tiroideo

sintetizan

las

hormonas

tiroxina

y

221.- para confirmar el diagnostico, deberá solicitar: a) determinación de niveles sericos de la hormona paratifoidea b) determinación de niveles sericos de hormonas tiroideas c) determinación de niveles servicios de catecolamina d) determinaciones sericas de cortisol e) tomografía axial computarizada de tórax 222.- los hallazgos de laboratorio que confirman el diagnostico son: a) calcio elevado b) T3 y T4 normal c) cortisol aumentado d) parathormona elevado e) T3 y T4 y capacitación de yodo radioactivo aumentados 223.- el tratamiento de estos enfermos, se hace básicamente con uno de los siguientes a) ansiolíticos b) glucocorticoides c) levotirosina d) triyodotironina e) propitiluracilo

Caso 54 Se presenta a consultorio un hombre de 20 años de edad con disuria, hiperbaralgesia lumbar, leucocitosis y bacteuria. La tension arterial registra 120/80 mmHg, frecuencia cardiaca de 88 por minuto y frecuencia respiratoria 22 por minuto. 224.- el diagnostico de mayor probabilidad es: a) Uretritis b) Glomerulonefritis c) Pielonefritis d) Prostatitis e) Litiasis renouretral 225.- la cantidad de bacterias / mililitro que en el examen general de orina es indicativo de infección de vías urinarias es: a) 100 b) 1000

34

c) 10 000 d) 100 000 e) 1 000 000 226.- debe hacerse diagnostico diferencial con las siguientes enfermedades menos con: a) nefrocalcinosis b) colecistitis c) pancreatitis d) tuberculosis genitourinario e) diverticulitis 227.- el tratamiento mas adecuado en este caso es: a) ingesta de abundantes líquidos b) prescripción de analgésicos c) suspender la activad sexual d) prescripción de fenazopiridina e) terapia antibioticoterapia en un mes 228.- de no contar con urocultivo ni antiobiograma, el tratamiento de elección es: a) cefazolina b) trimetropim con sulfametoxazol c) cefalexina d) cloramfenicol e) ampicilina Caso 55 Paciente masculino de 60 años, obeso, con amnesia de hechos recientes, irratibilidad, soliloquios y lenguaje incoherente. Su frecuencia cardiaca es de 90 por minuto y su presión arterial de 180/110 mmHg. El fondo de ojo muestra retinopatía hipertensiva. Durante su estudio se encontró colesterol serico de 400 mg con triglicéridos normales. 229.- el diagnostico más probable es: a) enfermedad de Reynud b) tromboangitis c) arteriosclerosis d) lupus eritematoso sistémico e) dermatomiositis 230.- lo más probables es que este paciente tenga una hiperlipoproteinemia de tipo: a) I b) II A c) II B d) III e) IV

Caso 56 Paciente de 45 años de edad con estudios de gabinete normales. Su BH resulta normal excepto de la bilirrubina fue de 2.4 mg. La deshidrogenasa láctica mayor de 2.00 UI y el coprorparasitoscopio presento giardia lamblia y grasas sin digestión total. 231.- el diagnostico probable del paciente es: a) carcinoma de colon ascendente b) pelagra pura c) sprue no tropical d) infección por virus de la inmunodeficiencia adquirida e) enfermedad de Crohn 232.- la fisiopatología de la anemia de este paciente sea debida a: a) aumento de requerimientos de folatos por neoplasia maligna b) deficiencia de acido nicotínico c) deficiencia de vitamina B12 por terapéutica anti VIH

35

d) deficiencia de folatos por mala absorción intestinal e) deficiencia del acido clorhídrico 233.- el origen de la anemia se identificaría solicitando la siguiente prueba de laboratorio: a) niveles de antígeno carcionoembriogenico en sangre b) niveles de triptano urinario c) cuantificación de linfocitos T CD4 d) medula ósea

Caso 56 Paciente femenina de 22 años de edad ama de casa que desde hace 1 año presenta lesiones en palma de manos y dedos construidas de vesículas, ampollas y eritema con gritas sangrientas. No tiene antecedentes familiares ni patológicos de importancia. Las lesiones producen prurito y dolor. 234.- la etiología del padecimiento es: a) hipersensibilidad retardada b) manifestación de atopia c) irritante primario d) desconocida e) leishmaniasis 235.- el diagnostico se confirma con: a) inmunodeficiencia ELISA b) biometría hematica c) reacción de montenegro d) pruebas del parche e) determinación de Ig E 236.- el diagnostico de este caso es: a) escabiosis b) dermatitis por contacto c) queratodermia palmo-plantar d) dermatitis atopica e) prurigo palmar 237.- la complicación mas frecuente de este padecimiento es: a) piodermitis b) liquenificacion c) eritrodermia d) epidermioliosis ampollar difusa e) formación de IDES

Caso 57 Se trata de un paciente del sexo masculino con 4 años de edad, proveniente de una familia que habita en una región marginada en la ciudad de México. A la exploración física se presenta una placa macular eritematosa y papuloescamosa con bordes activos delimitados por pequeñas vesículas, costras que evidencian huellas de rascado. La placa esta situada en el cuello y parte posterior de la oreja izquierda. Hasta el momento de la consulta el paciente no había recibido tratamiento. 238.- con los datos obtenidos en el cuadro clínico, el diagnostico más probable es: a) dermatitis atopica b) psoriasis c) prurigo por insectos d) micosis superficial e) impétigo húmedo

36

239.- el agente etiológico más común en esta dermatosis es: a) ácaros b) estreptococo beta hemolítico c) trichophyton rubrum d) herpes simple e) malassezia furfur 240.- por tanto, el manejo y tratamiento adecuado consiste en indicar la aplicación de: a) vioformo al 3% mas yodo en solución al 0.5% locales b) pomada con corticoesteroides mas yodo en solución al 0.5% c) penicilina procainica IM mas vioformo al 3% local d) griseofulvina oral mas yodo local en solución al 0.5% e) sulfas locales y antihistamínicos orales Caso 58 Paciente masculino de 6 meses de edad traído por presentar desde hace 2 semanas irratibilidad, llanto e inquietud. La madre niega que haya presentado fiebre, tos o diarrea. Se ha notado en regio glútea y genitales presencia de eritema, papulas y vesículas erosionadas. Se le aplico durante 5 días “synalar” con lo que disminuyeron las molestias, pero volvieron hace tres días al suspender el medicamento. 241.- el agente agresor más probable en este padecimiento es: a) acido úrico b) amoniaco c) enterobacterias d) picadura por insectos e) hongos 242.- el manejo inicial debe ser: a) acidificar la orina con salicilatos b) antibióticos sistémico de amplio espectro c) antibiótico tópico contra gramnegativos d) lavado con jabón neutro 2 veces al día e) suspender el medicamento que se ha usado 243.- si se pretende modificar el Ph del medio, debe prescribir: a) alquitrán de hulla b) aplicación de oxido de zinc c) aplicación de tópicos inertes d) acidificar el medio e)loción de calamina 244.- la complicación más frecuente de este cuadro se debe a: a) candida albicas b) escherichia coli c) uso de Synalr simple 245.- suspender los irritantes primarias para la cual: a) psoralenos b) nitrofuranos c) mercuriales Caso 59 Paciente femenina de 3 años de edad traída por iniciar ayer con cuadro caracterizado por lesiones pruriginosas en región lumbosacra, glúteo y abdomen, distribuidas en hileras pares. Ha presentado también tos seca ocasional y fiebre no cuantificada. Se refiere uno de sus hermanos inicia hace una semana con un cuadro similar, con nuevas lesiones ese día A la exploración física peso de 14.200 Kg, talla de 85 cm, temperatura de 37.2º C. se observa algunas lesiones como las escritas y otras decapitadas, con costras hematicas. 246.- el diagnostico más probable en este paciente es: a) escarlatina b) herpes zoster

37

c) alergia alimentaria d) prurigo por insectos e) exantema viral 247.- el agente causal seria: a) Cemex lectucularis b) estreptococo beta hemolítico c) rayos ultravioleta d) sarcoptes scabei e) toxinas por estafilococo 248.- un caso descrito al anterior puede ser producido por triatomas transmisores de: a) leishmania mexicana b) plasmodium ovale c) plasmodiux vivax d) tripanosoma gambiese e) tripanosoma cruzy 249.- el agente causante de: a) dengue hemorrágico b) enfermedad de chagas c) leishmaniasis d) malaria e) toxoplasmosis Caso 60 Paciente femenino de 10 años de edad sin antecedentes familiares de importancia. Hace 2 años padeció mononucleosis infecciosa. Desde hace 3 semanas noto aparición de placas alopécicas regulares en numero de 7 en total, de aproximadamente 1 cm de diámetro en región parietal, con superficie escamosa y puntos negros, que no originan molestias. El estado general es bueno.

250.- la impresión diagnostica del presente caso es: a) tiña tonsurante tricofitica b) tiña tonsurante microspora c) alopecia autoinmune d) tiña levota e) psoriasis 251.- el examen de las lesiones con luz de Wo rd en este caso mostrara: a) fluoresencia rojiza en la superficie b) fluorescencia verdosa en la superficie c) fluorescencia verdosa en el centro d) fluorescencia rojiza en el centro e) ninguna de las anteriores 252.- este proceso: a) no es contagiosa b) da lugar a epidemias escolares c) no afecta a los adultos d) es poco frecuente e) cura dejando alopecia residual definitiva 253.- el diagnostico diferencial se debe hacer con: a) dermatitis seborreica b) lupus eritematoso localizado c) queratitis pilar d) folículos atrofiante e) alopecia sifilítica 254.- el tratamiento de elección seria con: a) miconazol tópico

38

b) yodo al 0.5% tópico c) éter tópico al 2.3 % cada 24 hrs. d) griseofulvina VO 10/20 mg/Kg/día x 4-8 sem e) minoxidil tópico al 3% cada 24 horas Caso 61 paciente femenina de 24 años de edad, casada, enfermera, originaria de Culiacán, antecedentes hereditarios madre diabética controlada .antecedentes personales patológicos, cesaras por ruptura prematura de membranas Inicia su padecimiento hace 20 días con febrícula vespertina y cefalea ocasional de moderada intensidad y desde hace 15 días se asocia con mialgias generalizada, sudoración nocturna, fiebre de hasta 39º C por las tardes y adinamia vespertina y nocturna. A la exploración física TA 100/70 mmHg , 52Kg, frecuencia cardiaca de 72 por minuto , frecuencia respiratoria de 24 por minuto, temperatura de 39.7º C, buena hidratación 255.- los datos confirman el diagnostico de: a) dengue hemorrágico b) dengue clásico c) paludismo d) fiebre amarilla e) leptopirosis 256.- para confirmar el diagnostico pediría: a) biometría hematica b) pruebas serologicas c) determinación de complemento d) biometría hematica, factores de coagulación e) determinación de Ig E en suero 257.- el tratamiento de elección seria: a) acetominofen b) acido acetil salicílico c) Cinti inflamatorios no esteroideos d) corticoides e) medidas generales 258.- que medidas de prevención utilizaría en este enfermedad a) aislamiento de paciente de primer contagio b) eliminación de criadores c) eliminación de posibles fuentes de contaminación (fomites) Caso 62 El día 28 de Abril de 1993 una jurisdicción sanitaria del estado de Michoacán notifico por vía telefónica un probable brote de cólera en la comunidad X Por indignación inicial se entrevisto a las autoridades de salud, tanto estatales como  jurisdiccionales. Se supo que en dicho lugar el 19 de abril de 1993, ocurrió una defunción a consecuencia de un severo cuadro clínico caracterizado por diarrea y vomito y el sujeto fue sepultado dos días mas tarde. Se tomo previo permiso de los familiares, una muestra rectal del occiso con un hisopo impregnado de clayr blair, resultando positivo a vibrio cholerae El día 22 de abril se presentaron 3 casos mas con un cuadro clínico semejante a la de la persona fallecida. 259.- la prueba diagnostica del cholerae se realiza mediante: a) inmunodiagnostico b) clínica c) aislamiento e identificación del vibrion colérico en cultivos de heces y vomito d) brote epidémico e) análisis de laboratorio 260.- el caso descrito indica que el periodo de incubación del cólera es de: a) 6 horas b) 12 horas c) 35 a 48 horas

39

d) 72 horas e) 90 horas 261.- el tratamiento de elección para acortar el periodo de expulsión del vibrio cholerae 01 en las heces de estos enfermos es: a) trimetropim con sulfametoxasol b) dicloxacilina c) doxiciclina d) tetraciclina e) ampicilina 262.- para evitar la propagación de esta enfermedad, usted decide actuar sobre el reservorio natural del microorganismo, el cual se encuentra en: a) los animales b) el agua c) el hombre y el medio ambiente d) las deposiciones humanas y la basura e) las aves Caso 63 Paciente femenina de 11 años de edad, estudiante, sin antecedentes hereditarios ni patológicos de importancia. Desde hace 2 días con fiebre de 39º C, dolor abdominal difuso e intenso y evacuaciones liquidas en número de 8-10 en 24 horas de color verde con escaso moco y estrías sanguíneas. Estas molestias se acompañan de nauseas, vomito de contenido alimenticio en numero de 2 en 24 horas A la exploración física. Deshidratación leve, mal estado general con dolor abdominal en hipogastrio, signo del rebote negativo, aumento del peristaltismo intestinal en frecuente intensidad. 263.- la prueba diagnostica específica en este momento es: a) formula blanca b) coprocultivo c) hemocultivo d) reacciones febriles e) ninguna 264.- la impresión diagnostica de esta enfermedad es: a) amibiasis intestinal b) shigelosis c) salmonelosis d) giardiasis e9 intoxicacion alimentaria 265.- la complicación mas frecuente de esta enfermedad es: a) artritis b) perforación intestinal c) oclusión intestinal d) deshidratación e) ninguna 266.- el primer paso en el tratamiento de este caso seria: a) ampicilina b) trimetropim con sulfametoxasol c) difenoxilato con atropina d) eritromicina e) corrección de alteraciones de líquidos y electrolitos Caso 64 En una comunidad del estado de México de aproximadamente 10 000 habitantes, se notifico la probable presencia de 11 casos de tos ferina en niños menores de un año. Las cacteristicas clínicas que se reportaron fueron tos en accesos, fiebre de más de 38 grados, rinorrea y mal estado general.

40

267.- con esta información considera usted que se encuentra ante la presencia de: a) un brote b) una endemia c) una prevalecía d) una epidemia e) una notificación 268.- la tos ferina se caracteriza clínicamente por ser una enfermedad: a) de comienzo insidiosos que afecta el árbol traqueal con tos irritante que poco a poco se vuelve paroxística b) de comienzo insidioso que afecta el árbol traqueal, con tos irritante de predominio nocturno c) que Inicia con tos irritante que poco a poco se vuelve paroxística y es de predominio matutino d) que afecta el parénquima pulmonar, con tos paroxística, hemoptisis y ataque al estado general e) endémica, que afecta a los bronquios, con tos productiva que se concierte en paroxística y que afecta al estado genera 269.- entre las características epidemiológicas de la tos ferina encontramos que: a) todos los grupos de edad son susceptibles y su distribución mundial b) son más susceptibles los niños menores de 2 años c) la morbilidad y la mortalidad son mayores en las niñas que en los niños d) la tasa de letalidad es mayor en adultos que en niños menores de un año e) la morbilidad y la mortalidad son mayores en los niños que en las niñas 270.- su periodo de incubación es de: a) 3 a 5 días después del contacto b) 7 días y nunca excede los 21 días c) 10 días y nunca excede los 15 días d) 13 días después del contacto y nunca excede los 25 días e) 15 días después del contacto y nunca excede los 20 días 271.- los exámenes de laboratorio con que apoyaría el diagnostico clínico seria: a) biometría hematica y química sanguínea b) examen general de orina y química sanguínea c) exudado faringeo, biometría hematica con recuento de linfocitos d) hemocultivo y exudado faringeo e) hemocultivo, exudado faringeo y biometría hematica 272.- una medida específica que aplicaría para evitar la propagación de casos seria la aplicación de vacuna DPT a toda la población susceptible de 2 meses a: a) 1 año b) 2 años c) 4 años d) 6 años e) 5 años Caso 65 En una localidad de la ciudad de monterrey, nuevo leon, representativa de un estrato social bajo, se reportaron 4 casos de una enfermedad de inicio súbito. Un epidemiólogo realizo visita domiciliaría a todos los casos y encontró que cursaron con los siguientes signos y síntomas: fiebre de mas de 5 días de duración, cefalalgia intensa, dolores retroorbitales, articulares y musculares intensos, erupción; un caso se presento eritema generalizada 273.- se realizo el diagnostico de: a) varicela b) sarampión c) rubéola d) dengue e) paludismo 274.- el agente causal de:

41

a) un virus b) una bacteria c) un protozoario d) un bacilo e) una riquetsia 275.- es el mosquito identificado como el primer vector de la enfermedad: a) Aedes aeghypti b) anhopeheles c) simulido d) triatomas e) chince besucona 276.- el tipo de fiebre que se presenta en esta enfermedad: a) matutino vespertino b) vespertino c) matutino d) matutino vespertino e) a la media noche 277.- el periodo de incubación del agente causal, después de la picadura del mosquito es: a) 1-8 hrs b) 2-4 hrs c) 10-12 hrs d) 12-18 hrs e) 18-24 hrs 278.- ¿que otro reservorio se ha identificado al mosquito? a) complejo animal domestico b) complejo mosquito-mosquito c) complejo hombre-mosquito 279.- el tratamiento es a base de: a) acetominofem,. Analgésicos, antibioticoterapia b) analgésicos y antibióticos del grupo de las sulfamidas c) antivirales d) control de la fiebre (medios físicos, reposos y analgésicos del tipo de acetominofem) 280.- Cual estudio diagnostico en la enfermedad aguda de la enfermedad? a) cuantificación de antígeno vírico por inmunoflouresecencia b) inmunoenzimas (ELISA) c) biometría hematica., factores de coagulación 281.- las variables del macro ambiente que determinan la intensidad de la trasmisión son: a) la temperatura, la precipitación pluvial y la humedad b) el tipo de construcción de la vivienda y los hábitos de higiene c) el clima caluroso, al altura, los componentes del aire y la precipitación pluvial d) la temperatura calurosa y la inadecuada de las viviendas e) la edafología, la falta de ventilación y el hacinamiento

Caso 66 Paciente masculino de 3 años de edad con dermatosis de 6 días de evolución diseminadas en cara, tronco, raíces de miembros toracicos y pélvicos: inicialmente caracterizada por maculopapulas eritematosas, poco pruriginosas confluentes digitopresion (+) y posteriormente maculas hipercromicas residuales con descamación leve. Desde hace 10 días presenta fiebre que al principio fue elevado y persistente por de 3 días a la fecha disminuyo a 38º C. se ha acampando de malestar general, conjuntivitis con fotofobia

42

y tos seca. Se observa líneas de stimpsom en el borde de los parpados inferiores y hay adenomegalias cervical levemente dolorosa. 282.- con estos datos el diagnostico probable es: a) rubéola b) exantema súbito c) sarampión d) eritema infeccioso e) escarlatina 283.- los padres del menor le solicitan información acerca del periodo de contagiosidad de este exantema, ya que el paciente tiene un hermano menor de 1 año de edad le contesta que: a) 7 días antes del exantema a 5 días después del mismo b) del periodo de incubación hasta 3 días después del exantema c) 4 días antes del exantema a 3 días después de la primera lesión cutánea d) solo en la fase productiva e) se desconoce 284.- ¿Cuáles son las complicaciones de esta enfermedad? a) neumonitis b) bronquiolitis c) crisis convulsivas d) otitis media e) encefalitis Caso 67 Una mujer de 17 años de edad acudió a consulta con su medico por presentar malestar general de tres semanas de duración, el cual consiste en fiebre y dolor de garganta. La exploración física se observo congestión faringea, amígdalas con placas de exudado blanquecino y crecimiento ganglionar en cadenas cervicales posteriores, axilar e inguinal, hígado se palpo 1 cm del borde costal y el bazo a 2 centimetros. La Hb fue de 14 000 plaquetas de 250 000, leucocitos 7000, con 85 % linfocitos de los cuales el 85% fueron grandes y con vacuolas en el citoplasma 285.- la causa de esta enfermedad probablemente es: a) estreptococo beta hemolítico del grupo A b) virus de Epstein Barr c) estafilococo dorado d) neoplásica e) un coshakie-virus 286.- el mejor procedimiento para corroborar el diagnostico es: a) estudio de la medula ósea b) cultivo del exudado faringeo c) biopsia de un ganglio linfático d) reacciones febriles e) investigación de anticuerpos heterofilos 287.- el tratamiento de elección de este paciente debe hacerse a base de: a) prednisona b) tratamiento de sostén c) antibióticos

Caso 68 Paciente masculino de 4 años de edad que presenta desde meses antes disminución del apetito abdominal ocasional y episodios de diarrea, alterados con periodos asintomático. Ala exploración física se encuentra sucia, apática, pálida, bien hidratado, adelgazado, con

43

abdomen prominente. Talla 98 cm, 13.900Kg. Su diagnostico de presunción es parasitosis intestinal: 288.- usted decide confirmar agente causal es:

el diagnostico por el método de Gram., si sospecha que el

a) enterococus granulosos b) enterovios vermicularis c) strongyloides stercoralis d) taenia saginata e) trichuris trichuria 289.- identifique la helmantiasis transmitida por el suelo: a) filariasis b) oncocercosis c) teniasis d) triquinosis e) uncinariasis 290.- la cisticercosis se adquiere por ingerir: a) larvas de taenia b) t. saginata c) h. nana d) c. celulosae e) t. solium 291.- sus formas larvarias atraviesan la membrana alveolocapilar y producen el síndrome de Leitter: a) ascaris lumbricoides b) eschinococo granulosos c) teniasis d) entamoeba histolytica e) oncocercosis 292.- es el agente causal de la hidatidosis: a) enchylostoma doudenale b) capilaria hepatica c) eschinococus granulosos d) mansonella orzardi e) necator americanus 293.- parasitosis causal de anemia microcita: a) enchilostoma doudenale b) capilaria hepatica c) eschinococs granulosos d) masonella orzardi e) necator americans 294.- parasito causante del prolapso rectal: a) onchocerva volvulos b) toxicar hominis c) tricdhuris trichuria d) trichinella spiralis e) wucherona branconi

Caso 69

44

paciente masculino de 8 años de edad originario de una región tropical rural que acude a la consulta por presentar dolor abdominal tipo cólico en fosa iliaca derecha acompañado de evacuaciones semipastosas con huellas de sangre f resca. A la exploración física se observa abdomen globoso, mucosa conjuntival pálida. 295.- señale el diagnostico del caso anterior: a) amibiasis intestinal crónica b) giardiasis crónica c) teniasis d) oxiriosis e) ascariosis 296.- la triquinosis es ocasionada por: a) ptahelmintos b) protozoarios c) hemathelmintos d) bacterias e) ricketsias 297.- su forma de transmisión es por: a) via directa de persona a persona b) por la ingestión de carne cruda o mal cocida preferentemente de suine c) consumo de agua sin hervir ni clorar d) contacto con excremento de animales domésticos como el cerdo, patos, perros e) por las vías aéreas 298.- el periodo de incubación de la triquinosis va de: a) 1-5 días b) 6-10 días c) 11-15 días d) 3-4 semanas e) 1-2 semanas 299.- en México el agente infeccioso causante de la triquinosis es triquinilla: a) spiralis nativa b) spiralis Nelson c) cellulosae d) spiralis e) pylori 300.- el diagnostico se facilita en ocasiones por la identificación en sangre de: a) eosinofilos y por datos de pruebas serologicas 301.- los animales domésticos reservorios del agente causal son: a) pollos, bovinos y patos b) cerdos, perros y gatos c) peces, gatos y pollos d) ratas, pollos y peces e) caballos, vacas y asnos 302.- el tratamiento específico se realiza con base a la administración de: a) metronidazol b) trimetropim con sulfametoxazol c) metronidazol d) ampicilina e) ceftazidima 303.- en este padecimiento la medida preventiva primaria es: a) control de roedores b) reglamentación sanitaria para matanza y crianza de ganado c) lavarse las manos después de ir al baño y antes de comer d) floración del agua e) control de excretas de animales

45

Caso 70 Paciente masculino de 62 años de edad originario de guerrero, campesino, viudo, fumador de 20 cigarrillo al día, bebe dos litros de pulque diariamente, amputado de dos dedos de la mano izquierda en forma accidental hace 10 años. Al examen físico se observa que tiene sus piezas dentales en mal estado de aseo con múltiples caries. En el pie derecho hay aumento de volumen con una placa visible pero palpable, adherida a planos profundos, dura de aproximadamente 3 cm con salida de liquido purulento. 304.- confirma el diagnostico de: a) micetoma b) esporotricosis c) complejo vascular de pierna d) linfadenopatia e) estasis venosa 305.- el agente causal de esta etiología es: a) nocardia brasilienses b) granulos negros c) s. shuencky d) impétigo secundario e) estasis venosa 306.- el diagnostico se confirma mediante: a) biopsia de ganglio linfático b) antibiograma y cultivo c) luz de Word d) cultivo de líquido purulento Caso 71 Se atiende recién nacido el cual presenta asindactilia, labio y paladar hendido 307.- usted considera que el diagnostico más probable es: a) poliuvirus b) rubéola congénita c) síndrome de turner d) toxoplasmosis congénita e) enfermedad citomegalica 308.- la prueba de laboratorio que confirmaría su diagnostico es: a) tinción de Gram. b) perfil cromosómico c) cultivo intracelular d) presencia de Ig M especifica en el recién nacido e) búsqueda de oocistos en coproparasitoscopios 309.- las medidas preventivas recomendables para impedir la transmisión de la enfermedad son: a) evitar el consumo de carne cruda y eliminación adecuada de las heces de animales domésticos b) inmunización activa y evitar la promiscuidad sexual c) evitar el consumo de carne de cerdo y pollo d) vacunar a los animales e) inmunización activa y extremar las medidas higiénicas Caso 72.Paciente masculino de 34 años de edad, es admitido en el hospital por presentar obnubilación mental y asterixis de un día de evolución. Es un alcohólico crónico que ingirió bebidas embriagantes en abundancia el día anterior. A la exploración física se observa ictericia, angiomas en araña, red venosa colateral en el abdomen, ginecomastia, ascitis, edema blando en piernas, melena y vello pubiano escaso

46

310.- la etiología de los trastornos neurológicos del paciente es: a) encefalopatía alcohólica b) amibiasis intestinal crónica c) encefalopatía hipertensiva d) crisis convulsivas e) tumor a distancia 311.- el mecanismo más importante del edema es: a) aumento en la producción de hormon a antidiurética b) hiperaldosteronismo secundario c) reabsorción proximal aumentada de sodio en el riñón d) obstrucción de la vena cava inferior e) insuficiencia venosa de miembros inferiores 312.- una derivación porto-sistémica en este paciente producira: a) mejoría del edema de miembros inferiores b) disminución de la frecuencia de encefalopatía c) mejoría de la circulación arterial hepatica d) disminución de la hipertensión portal e) mayor sobrevira Caso 73 Paciente masculino de 71 años de edad narra que l0 meses atrás presento una lesión microscópica, ante la inquietud y el temor recurrió a un compañero quien le recomendó tomar un analgésico con antiséptico urinario. Hace 4 meses se día atención por notar franca disminución de la fuerza del chorro urinario y dolor en la parte inferior del abdomen. El dolor se exacerba a la palpación. 313.- estas manifestaciones clínicas pueden atribuirse a : a) tumor de ampula rectal b) tumor de origen prostático c) pielonefritis d) litiasis vesicoureteral e) litiasis renal 314.- el tipo de lesión que se presenta con más frecuencia en pacientes de esta edad es: a) carcinoma de próstata b) carcinoma de vejiga c) HPB d) carcinoma de pene e) pielonefritis 315.- para mejorar la sintomatología el procedimiento que usted recomendaría es: a) dilatación uretral, talla vesical de elección y cateterismo vesical a derivación b)m dilatación uretral y talla vesical de elección c) dilatación uretral y biopsia de próstata d) cateterismo vesical a derivación con dilatación uretral y biopsia e) talla vesical Informe histopatológico conglomerados de glándulas pequeñas y grandes de epitelio cúbico y membrana basal intacta, dispuesta en yemas papilares que contiene secreción espesa y escasos cúmulos de linfocitos 316.- con esta descripción usted corrobora el diagnostico de: a) prostatis inflamatoria inespecífica b) hipertrofia prostática benigna c) carcinoma de próstata d) prostatis granulomatosa e) carcinoma de vejiga

47

Caso 74 paciente masculino de 578 años de edad quien ingresa a urgencias inconciente con cianosis labial pupilas dilatadas, arreflexico, la tension arterial no audible, aparato respiratorio presenta periodos de apnea, la temperatura de 35º C y el trazo electrocardiográfico corresponde a fibrilación ventricular 317.- la sobrevida del paciente depende de: a) intubación orotraqueal b) desfibrilacion rápida c) punción subclavia d) monitoreo cardiaco e9 arritmia ventricular 318.- la dosis de adrenalina a administrar es de: a) 0.1 mg/ Kg b) 1 mg/ Kg c) 10 mg/Kg d) 100 mg/Kg e) 1 g/Kg 319.- el tiempo en que debe administrar adrenalina IV es: a9 infusión continúa b) cada minuto c) cada 5 minutos d) cada 10 minutos e) cada 15 minutos 320.- el inotropico de elección en los cuidados de postreanimacion es: a) adrenalina b) noradrenalina c) isoproterenol d) dopamina e) disoxina Caso 75 paciente masculino de 55 años de edad que acude refiriendo ingesta de bebidas alcohólicas desde hace 2 días, hoy, posterior a la ingesta de abundantes alimentos grasosos, presenta dolor abdominal en epigastrio, irradiado a ambos hipocondrios, trasnfictivo e intenso, acampando de estado nauseoso y vomito en 5 ocasiones A su llegada se encuentra con presión arterial de 100/70 mmHg, frecuencia cardiaca de 120 por minuto, frecuencia respiratoria de 25 por minuto, temperatura de 38º C, pálido, diaforético, en posición de gatillo, con escasos estertores subcrepitantes bibasales. El abdomen se encuentra distendido con hipoperistalsis, dolor a la palpación profunda en mesogastrio. 321.- con los datos obtenidos establecerá el diagnostico de: a) pancreatitis b) pancreatitis hemorrágica c) carcinoma de cabeza de páncreas d) tumor de células delta de páncreas e) coledocolitisis con infección agregada 322.- el dato de laboratorio que confirmaría el diagnostico es: a) amilasa b) lipasa c) amilasa y lipasa d) transaminasa glutamica piruvica e) transaminasa glutamica oxalacetica 323.- el analgésico de elección en este padecimiento es: a) metamizol sodico b) morfina

48

c9 diclofenaco sodico d) meta dona e) meperidina 324.- una semana mas tarde el paciente que persiste con dolor abdominal, fiebre, leucocitosis,, masa palpable en epigastrio. La sospecha clínica es: a) pseudoquiste b) piocolecisto c) flemón difuso d) perforación de absceso hepático e) neoplasia 325.- la vigilancia de este paciente debe llevarse a cabo en: a9 terapia intermedia b) sala de urgencias c) consulta externa d) terapia intensiva e9 unidad metabólica Caso 76.- paciente de 22 años de edad con cuadro caracterizado por una marcada regresión a una conducta primitiva-deshuibida y desorganizada. Tiene el pensamiento muy alterado y su contacto con la realidad es muy deficiente. Su apariencia personal y su conducta social se hayan destruidas. Sus respuestas emocionales son inadecuadas y frecuentemente estalla en carcajadas inapropiadas o sin razón aparente. Su conducta se describe como tonta o fatida. 326.- este cuadro se refiere a : a) trastorno bipolar tipo maniaco b) epilepsia de lóbulo temporal c) retardo mental medio d) esquizofrenia tipo desorganizado o hebrefrenica e) catatonia excitada 327.- para el diagnostico diferencial deben considerarse los siguientes factores menos uno: a) depresión analítica b) transtorno afectivo bipolar c) trastorno delirante paranoide d) transtorno mental orgánico e) síndrome delirante orgánico (anfetaminas o feniclidina) 328.- en este cuadro clínico usted no esperaria encontrar: a) ideas delirantes b) alucinaciones predominantes c) fobia hacia lo nuevo d) afectividad embotada o inapropiada e) incoherencia 329.- donde actúan los fármacos dopaminergicos Caso 77 Paciente masculino de 45 años de edad que refiere dolor en epigastrio intenso, ardoroso, que le provoco desmayo lo cual sus acompañante, este relato que el paciente manifestó tener agruras con frecuencia por lo cual tomo antiácidos. A la exploración física se le encontró suporoso, diaforético, pálido, con tensión arterial de 90/40 mmHg y pulso débil de 120 por minuto. Había taquicardia sinusal y los pulmones. El abdomen era blandos, depresible sin visceromegalias. Con aumento de sus peristalsis. En región perianal había huellas de sangre oscura 330.- ¿Cuál de los siguientes es el mejor para confirmar el diagnostico en este momento? a) radiografía simple de abdomen b) serie esófago gastro duodenal c) arteriografía selectiva d) endoscopia esófago-gastro-duodenal e) electrocardiograma

49

331.- el tratamiento deberá incluir la administración de : a) vasodilatadores coronarios b) anticolinergicos vía parenteral c) sangre total d) concentración globular e) plasma 332.- si después de 6 horas de iniciado el manejo hospitalario persiste la hipotensión arterial. El tratamiento de elección es: a) laparotomía b) lavado gástrico c) administración de digitalices d) administración de norepinefrina IV e) administración de dopamina Caso 78 Un paciente masculino en edad escolar es llevado al medico por su madre, quien refiere hemorragia profusa por la nariz en dos ocasiones, las cuales se presentaron después de limpiarse la nariz. Niega antecedentes patológicos de importancia para su padecimiento. A su ingreso los signos vitales fueron normales. A la exploración física se observa huellas de sangrado activo en la nariz izquierda, mucosa nasal hiperemica sin secreción muicopurulenta. Niega equimosis por microtraumatimos lo cual es confirmado en la exploración. Se realiza presión digital a nivel del área septal durante 5 minutos produciéndose inhibición del sangrado. 333.- el diagnostico más probable del caso anterior es: a) epistaxis b) purpura c) rinoescleroma d) rinitis seca e) fiebre del heno 334.- la etiología más probable en este caso es: a) traumática b) alérgica c) bacteriana d) viral e) micotica Caso 79 Acude a consulta un niño de 8 años de edad el cual presenta dolor en oreja derecha hace 5 días después de un traumatismo directo que sufrió en la oreja derecha, con signos vitales dentro de la normalidad. Al examen físico se encuentra bien hidratado, con oreja derecha edematizada, dolorosa, caliente, con proceso infeccioso y que no afecta la audición, dolor en región mastoidea a la ligera palpación, el resto del examen fisico normal. 335.- con los datos obrtenidos considera el diagnostico de: a) otitis media serosa b) otitis externa c) otitis media d) otosclerosis e) sinusitis 336.- el agente etiológico con más frecuencia es: a) streptococo beta hemolítico del grupo A b) streptococo beta hemolítico del grupo B c) estafilococo dorado d) pseudomona aeroginosa e) herpes virus 337.- el tratamiento de elección es:

50

a) medidas generales b) drenaje del absceso c) cefalosporinas de tercera generación d) azitromicina e) penicilina 338.- el niño mejoro considerablemente a las 72 horas de su8ministrar el tratamiento sin embargo se observa tumefacción en la base de la oreja derecha, por lo que usted decide: a) únicamente cambiar el antibiótico b) completar el esquema del tratamiento c) adicionar antiinflamatorios y completar 10 días del tratamiento d) cambiar el antibiótico y adicionar antiinflamatorios por 7 días e) únicamente suministrar antiinflamatorios Caso 80 Un niño de 8 años de edad, conocido como asmático desde los dos años con crisis que ameritan hospitalización hasta 5 veces por año, presenta síntomas intensos por las noches con el ejercicio 340.- en este niño, la mejor forma de medir el compromiso del fluido aéreo será: a) volumen residual y flujo espiratorio máximo b) volumen residual y capacitación vital c) capacidad vital forzada y volumen espiratorio forzado d) flujo espiratorio máximo e) capacidad vital 341.- en el protocolo de estudio inicial de este niño ¿Cuál no es una prueba diagnostica apropiada? A) citología hematica b) rx de tórax c) citología nasal d) pruebas cutáneas e) rx de senos paranasales 342.- en caso de una crisis asmática moderada el manejo inicial en la sala de urgencias debe ser: a) aminofilina IV b) salbutamol inhalado c) terbutalina subcutánea d) hidrocortisona IV e) oxigeno solamente 343.- usted debe instalar manejo farmacológico profiláctico, el agente mas útil es: a) teofilina VO b) betametasona inhalada c) prednisona IV d) salmeterol inhalado e) cromoglicato disodico inhalado Caso 81 Paciente femenino de 6 años y medio de edad que desde hace 3 días presenta fiebre de 38º C y dolor abdominal bajo, ayer se agrego vomito enana ocasión y adolerimiento en región lumbar. No hay datos de toxicidad en este momento y usted decide manejarlo ambulatoriamente. 344.- el diagnostico más probable es: a) colitis aguda b) infección de vías urinarias c) tumor de wilms 345.- el tratamiento de elección seria: a) antibióticos b) antiinflamatorios

51

c) antisépticos urinarios

Caso 82 Un niño de 5 años de edad presenta en los últimos 8 días edema que inicio en parpados, luego se generalizo. Hay oliguria pero no importante y tiene una infección respiratoria agregada. La exploración física corrobora el edema generalizado y los signos vitales son norm ales. En el laboratorio se descubre hipoalbulinemia aumento de colesterol serico, proteinuria masiva y en el examen de orina aparecen 3 eritrocitos por campo. En el primer episodio de este tipo en el paciente. 346.- el diagnostico de este paciente es: a) síndrome nefrotico de cambios mínimos b) síndrome nefrotico por antitoxinas c) infección urinaria alta. Pielonefritis d) glomerulonefritis membranosa e) amiloidosis renal 347.- de las complicaciones siguientes ¿Cuál no es frecuente en estos niños? a) bronconeumonia b) gastroenteritis c) peritonitis primaria d) erisipela e) litiasis renal 348.- el manejo mas adecuado en este paciente seria: a) restricción de líquidos solamente b) restricción de líquidos y terapia renal c) diuréticos y ciclosfofamida d) esteroides por 4 semanas y restricción de líquidos e) esteroides por 8 semanas y restricción de líquidos Caso 83 Paciente del sexo masculino de 32 años de edad solicita atención médica por presentar rinorrea, estornudos y congestión nasal así como dolor de garganta. Ala exploración física los signos vitales se observan dentro de los límites normales y en orofaringe se aprecia la existencia de congestión faringea. El recuento leucocitario y la eritrosedimentacion son normales 349.- la causa etiológica más frecuente es: a) retrovirus b) rinovirus c) parvovirus d) adeniovirus e) virus gripal A 350.- el tratamiento consiste en la administración de: a9 antiinflamatorios y antihistamínicos b) antibióticos y antipiréticos c) antibióticos y antihistamínicos d) analgésicos y descongestionantes e) broncodilatadores y antivirales 351.- para evitar la propagación de la enfermedad usted recomendaría: a) aislamiento del paciente b) ambiente húmedo 352.- usted considera que el riesgo estimado para el uso previo de hormonas exógenos y cáncer de endometrio fue: a) 0.2

52

b) 0.5 c) 2.0 d) 3.0 e) 5.0 353.- podría concluirse de acuerdo con los datos del estudio: a) no existe entre el uso de hormonas exógenos y cáncer de endometrio b) no podemos hacer inferencias causales ya que existe el mismo número de mujeres con cáncer de endometrio que en mujeres sanas c) no podemos hacer inferencias causales ya que no tenemos el mismo número de mujeres con cáncer de endometrio que en mujeres sanas. d) no podemos hacer inferencias causales ya que no tenemos el mismo número de mujeres expuestas que no expuestas e) existe una aparente asociación entre el uso de hormonas erógenas y cáncer de endometrio Caso 84.Paciente femenina de 25 años de edad que acude a consulta para orientación medica sobre el método de anticoncepción que habrá de elegir. Sus antecedentes son: menarca a los 13 años, ritmo 30X5 eumenorreica, inicia VSA a los 20 años , gesta 2, para 2. Exploración física general y especuloscopia normal 354.- con estos antecedentes ¿Qué método anticonceptivo es el mas indicado en esta paciente? a) anticonceptivos hormonales orales b) anticonceptivos hormonales inyectables c) dispositivo intrauterino d) anticonceptivos locales e) oclusión tubaria bilateral 355.- después de la administración o colocación del método anticonceptivo elegido, diga que síntomas o signos puede presentar regularmente la paciente: a) cloasma y porimenorragia b) dolor cólico abdominal y hemorragia transvaginal c) hiperestesia e hiperbaralgesia abdominal d) cefalalgia y aumento de peso e) cefalalgia e irratibilidad 356.- con el método anticonceptivo que eligió ¿Qué complicación importante se puede presentar? a) embolia pulmonar b) hipertensión arterial c) perforación uterina d) reacción alérgica local e) cloasma Caso 85

Una paciente de 88 años de edad, con cardiopatía mixta (hipertensiva y arterioesclerótica), se interna para tratamiento quirúrgico de fractura intertrocantérea izquierda. Durante su internamiento se administraron soluciones parenterales y dieta; no se llevó control de líquidos.  A las 24 horas de su internamiento presenta disnea de reposo, ansiedad, inquietud y diaforesis; durante la exploración física se le encontró congestión pulmonar, tensión arterial de 110/60 mm Hg, pulso de 85 por minuto, temperatura de 36.5°C y respiración de 28 por minuto. 357. El diagnóstico inicial que se puede establecer con estos datos es: A) insuficiencia cardiaca derecha B) edema agudo pulmonar  C) expectoración de tipo biliar  D) expectoración asalmonada 358. Se solicita un estudio de telerradiografía de tórax en esta paciente, se espera encontrar:  A) hipertrofia ventricular derecha B) derrame pleural bilateral basal

53

C) infiltrado intersticial y perihiliar  D) imagen de condensación pulmonar  359. La causa más probable del padecimiento es:  A) sobrecarga de líquidos B) arterioesclerosis C) dilatación de la fibra miocárdica D) émbolos en la arteria pulmonar  Caso 86 Un paciente de 38 años acude a consulta por sufrir astenia, anorexia y náusea de dos meses de evolución, agregándose desde hace un mes dolor abdominal, impotencia sexual e ictericia leve. El enfermo refiere que ha perdido 4 kg de peso en ese lapso. Durante la exploración física, la tensión arterial es de 110/80 mm Hg, la frecuencia cardiaca de 82 por minuto, peso de 74 kg, talla de 1.69 m. Se observa ictericia conjuntival leve, caries múltiples, hipertrofia parotídea, tórax sin datos anormales. En el abdomen se aprecia una hernia umbilical pequeña y hay hemorroides. La biometría hemática reporta hemoglobina de 11 g/dl.

360. Es muy probable que el paciente tenga antecedentes de:  A) ingestión crónica de alcohol B) exposición a plaguicidas C) intoxicación por plomo D) intoxicación por anilinas 361. El laboratorio reporta: glucemia de 106 mg/dl; creatinina 1.2 mg/dl, leucocitos 4,500/mm 3, tiempo de protrombina alargado, hierro sérico normal, transaminasa glutámico oxalacética de 85 UI/L y albúmina de suero de 2.6 g/dl. 2. Con estos datos, el procedimiento más adecuado para el diagnóstico es: A) determinar TGP y TGO B) gammagrama de hígado C) ultrasonido abdominal D) biopsia hepática 362. Esta hemorragia se debe más probablemente a:  A) ruptura de várices esofágicas B) gastritis erosiva sangrante C) úlcera duodenal D) divertículo duodenal

Caso 87 Una niña de cinco años es llevada a consulta por presentar malestar general, hipertermia de 39 ° C, máculas eritrematosas e intensamente pruriginosa, localizadas en cuero cabelludo, cara, tronco y en las porciones proximales de las extremidades; sin crecimiento importante de ganglios retroauriculares. La madre informa que el día anterior la niña presentó ligero malestar  y febrícula, además de que por la noche se mostró inquieta y con algunos problemas para conciliar el sueño. 363. Con los datos proporcionados se puede hacer el diagnóstico siguiente:  A) sarampión B) rubéola C) varicela D) roseola 364. El agente etiológico causante de este cuadro es: A) paramixovirus B) herpes virus C) citomegalovirus D) rubivirus 365

¿Cuáles serían las medidas generales adecuadas para esta niña?

54

A) Recortar las uñas, lociones antipruriginosas y antihistamínicos B) Recortar las uñas, lociones antipruriginosas y expectorantes C) Antibacterianos, antihistamínicos y antipiréticos D) Antihistamínicos, antibacterianos y expectorantes

366. En caso de requerir un fármaco específico, ¿cuál sería el de primera elección?

A) Amantadina B) Aciclovir  C) Ganciclovir  D) Foscarnet Caso 88 Una primigesta de 16 años, con embarazo de diez semanas, ingresa al hospital por presentar  náusea y vómitos excesivos. Al examen físico el útero está agrandado, casi hasta el ombligo, y hay crecimiento ovárico bilateral. La paciente admite tener manchado hemático persistente. 367. El diagnóstico más probable es:  A) embarazo peritoneal B) mola hidatiforme C) miomatosis uterina múltiple D) embarazo ectópico roto

368. Una causa común de muerte en esta lesión es:  A) muerte súbita inesperada B) embolia pulmonar  C) obstrucción intestinal D) perforación y hemorragia uterina 369. El síntoma principal en este caso es:  A) dolor  B) útero agrandado C) sangrado D) náusea y vómito 370 Es común que en estos casos resulte:

A) pérdida del útero B) cáncer a largo plazo C) embarazo ectópico subsiguiente D) insuficiencia cervical Caso 89 Un paciente de 21 años, sin antecedentes previos de importancia, acude al servicio de urgencias por presentar sensación de fiebre, escalofríos y dificultad respiratoria. Refiere que desde hace cinco días existe dolor de mediana intensidad, el cual inició en epigastrio y posteriormente se irradió a la fosa ilíaca derecha. Esta molestia aumenta con la ingesta de alimentos y se acompaña de sensación de náusea. En la exploración física de detecta tensión arterial de 80/50 mm Hg, pulso de 120 por minuto, frecuencia respiratoria de 28 por minuto y temperatura de 38.5°C. A la palpación del abdomen se despierta dolor, encontrándose rigidez muscular y signo de rebote positivo en forma generalizada. 371 El cuadro clínico del paciente corresponde a: A) apendicitis subaguda B) sepsis e hipovolemia C) apendicitis retrocecal D) gastroenterocolitis La causa más probable del padecimiento es: A) apendicitis perforada

372.

55

B) trombosis mesentérica C) absceso por divertículo colónico D) vólvulo

373. El tratamiento de urgencia inicial será con base en: A) laparotomía exploradora B) restablecimiento de hemodinamia C) hidratación más anticolinérgicos D) antibióticos de amplio espectro Caso 90 El personal de vigilancia epidemiológica de los Servicios Coordinados de Salud en el estado de Jalisco reportó en 1996, para la semana epidemiológica número 36, un total de 203 casos de enfermedad respiratoria crónica; para la misma semana del año anterior sólo había notificado 99 casos. 374. Con los datos anteriores, usted pensaría que se encuentra ante: A) un brote B) una endemia C) una epidemia D) una incidencia 375 El diagnóstico más probable es:  A) cáncer broncogénico B) coccidiodomicosis C) histoplasmosis D) tuberculosis 376. Existen varios métodos diagnósticos para esta enfermedad. El más común es:  A) inmunoelectroforesis B) citología en esputo C) baciloscopía de esputo D) serología Caso 91 Se presenta a consulta un niño de 8 años de edad aparen tem,ebrte sano con presencia de gigantismo, caracterizado por miembros superiores mayor que inferiores con lordosis lumbar  sin otra sintomatología: 377.- con los datos obtenidos se hace el diagnostico: a) acondroplasia b) tetralogia de fallot c) rubéola d) síndrome de prader wuilson 378.- el carcter se determina geneticamente por: a) crmosoma ligado al X b) gen dominante liogada al X c) gen recesivo ligada al X 379.- el tratamiento a realizar es: Caso 92  Acude a consulta paciente femenina de 25 años de edad por que presento el dia de su regla tumoracion anuivel de mama derecha sin sintomatología ninguna.  Al examen fisico se encuetra tumor depresible movible sin dolor a la palpacion, sin otra sintomatología

56

View more...

Comments

Copyright ©2017 KUPDF Inc.
SUPPORT KUPDF